Preview (15 of 117 pages)

Preview Extract

ATI Pharmacology Proctored Test Bank
1. A nurse is caring for a client with hyperparathyroidism and notes that the client's serum
calcium level is 13 mg/dL. Which medication should the nurse prepare to administer as
prescribed to the client?
A. Calcium chloride
B. Calcium gluconate
C. Calcitonin (Miacalcin)
D. Large doses of vitamin D
Answer: C. Calcitonin (Miacalcin)
Rationale:
The normal serum calcium level is 8.6 to 10.0 mg/dL. This client is experiencing
hypercalcemia. Calcium gluconate and calcium chloride are medications used for the
treatment of tetany, which occurs as a result of acute hypocalcemia. In hypercalcemia, large
doses of vitamin D need to be avoided. Calcitonin, a thyroid hormone, decreases the plasma
calcium level by inhibiting bone resorption and lowering the serum calcium concentration.
2. Oral iron supplements are prescribed for a 6-year-old child with iron deficiency anemia.
The nurse instructs the mother to administer the iron with which best food item?
A. Milk
B. Water
C. Apple juice
D. Orange juice
Answer: D. Orange juice
Rationale:
Vitamin C increases the absorption of iron by the body. The mother should be instructed to
administer the medication with a citrus fruit or a juice that is high in vitamin C. Milk may
affect absorption of the iron. Water will not assist in absorption. Orange juice contains a
greater amount of vitamin C than apple juice.
3. Salicylic acid is prescribed for a client with a diagnosis of psoriasis. The nurse monitors
the client, knowing that which of the following would indicate the presence of systemic
toxicity from this medication?

A. Tinnitus
B. Diarrhea
C. Constipation
D. Decreased respirations
Answer: A. Tinnitus
Rationale:
Salicylic acid is absorbed readily through the skin, and systemic toxicity (salicylism) can
result. Symptoms include tinnitus, dizziness, hyperpnea, and psychological disturbances.
Constipation and diarrhea are not associated with salicylism.
4. The camp nurse asks the children preparing to swim in the lake if they have applied
sunscreen. The nurse reminds the children that chemical sunscreens are most effective when
applied:
A. Immediately before swimming
B. 15 minutes before exposure to the sun
C. Immediately before exposure to the sun
D. At least 30 minutes before exposure to the sun
Answer: D. At least 30 minutes before exposure to the sun
Rationale:
Sunscreens are most effective when applied at least 30 minutes before exposure to the sun so
that they can penetrate the skin. All sunscreens should be reapplied after swimming or
sweating.
5. Mafenide acetate (Sulfamylon) is prescribed for the client with a burn injury. When
applying the medication, the client complains of local discomfort and burning. Which of the
following is the most appropriate nursing action?
A. Notifying the registered nurse
B. Discontinuing the medication
C. Informing the client that this is normal
D. Applying a thinner film than prescribed to the burn site
Answer: C. Informing the client that this is normal
Rationale:
Mafenide acetate is bacteriostatic for gram-negative and gram-positive organisms and is used
to treat burns to reduce bacteria present in avascular tissues. The client should be informed

that the medication will cause local discomfort and burning and that this is a normal reaction;
therefore options 1, 2, and 4 are incorrect
6. The burn client is receiving treatments of topical mafenide acetate (Sulfamylon) to the site
of injury. The nurse monitors the client, knowing that which of the following indicates that a
systemic effect has occurred?
A. Hyperventilation
B. Elevated blood pressure
C. Local pain at the burn site
D. Local rash at the burn site
Answer: A. Hyperventilation
Rationale:
Mafenide acetate is a carbonic anhydrase inhibitor and can suppress renal excretion of acid,
thereby causing acidosis. Clients receiving this treatment should be monitored for signs of an
acid-base imbalance (hyperventilation). If thisoccurs, the medication should be discontinued
for 1 to 2 days. Options 3 and 4describe local rather than systemic effects. An elevated blood
pressure may be expected from the pain that occurs with a burn injury.
7. Isotretinoin is prescribed for a client with severe acne. Before the administration of this
medication, the nurse anticipates that which laboratory test will be prescribed?
A. Platelet count
B. Triglyceride level
C. Complete blood count
D. White blood cell count
Answer: B. Triglyceride level
Rationale:
Isotretinoin can elevate triglyceride levels. Blood triglyceride levels should be measured
before treatment and periodically thereafter until the effect on the triglycerides has been
evaluated. Options 1, 3, and 4 do not need to be monitored specifically during this treatment.
8. A client with severe acne is seen in the clinic and the health care provider (HCP) prescribes
isotretinoin. The nurse reviews the client's medication record and would contact the (HCP) if
the client is taking which medication?
A. Vitamin A

B. Digoxin (Lanoxin)
C. Furosemide (Lasix)
D. Phenytoin (Dilantin)
Answer: A. Vitamin A
Rationale:
Isotretinoin is a metabolite of vitamin A and can produce generalized intensification of
isotretinoin toxicity. Because of the potential for increased toxicity, vitamin A supplements
should be discontinued before isotretinoin therapy. Options 2, 3, and 4 are not contraindicated
with the use of isotretinoin.
9. The nurse is applying a topical corticosteroid to a client with eczema. The nurse would
monitor for the potential for increased systemic absorption of the medication if the
medication were being applied to which of the following body areas?
A. Back
B. Axilla
C. Soles of the feet
D. Palms of the hands
Answer: B. Axilla
Rationale:
Topical corticosteroids can be absorbed into the systemic circulation. Absorption is higher
from regions where the skin is especially permeable (scalp, axilla, face, eyelids, neck,
perineum, genitalia), and lower from regions in which permeability is poor (back, palms,
soles).
10. The clinic nurse is performing an admission assessment on a client. The nurse notes that
the client is taking azelaic acid (Azelex). Because of the medication prescription, the nurse
would suspect that the client is being treated for:
A. Acne
B. Eczema
C. Hair loss
D. Herpes simplex
Answer: A. Acne
Rationale:

Azelaic acid is a topical medication used to treat mild to moderate acne. The acid appears to
work by suppressing the growth of Propionibacterium acnes and decreasing the proliferation
of keratinocytes. Options 2, 3, and 4 are incorrect.
11. The health care provider has prescribed silver sulfadiazine (Silvadene) for the client with
a partial- thickness burn, which has cultured positive for gram- negative bacteria. The nurse is
reinforcing information to the client about the medication. Which statement made by the
client indicates a lack of understanding about the treatments?
A. "The medication is an antibacterial."
B. "The medication will help heal the burn."
C. "The medication will permanently stain my skin."
D. "The medication should be applied directly to the wound."
Answer: C. "The medication will permanently stain my skin."
Rationale:
Silver sulfadiazine (Silvadene) is an antibacterial that has a broad spectrum of activity against
gram-negative bacteria, gram-positive bacteria, and yeast. It is applied directly to the wound
to assist in healing. It does not stain the skin.
12. A nurse is caring for a client who is receiving an intravenous (IV) infusion of an
antineoplastic medication. During the infusion, the client complains of pain at the insertion
site. During an inspection of the site, the nurse notes redness and swelling and that the rate of
infusion of the medication has slowed. The nurse should take which appropriate action?
A. Notify the registered nurse.
B. Administer pain medication to reduce the discomfort.
C. Apply ice and maintain the infusion rate, as prescribed.
D. Elevate the extremity of the IV site, and slow the infusion.
Answer: A. Notify the registered nurse.
Rationale:
When antineoplastic medications (Chemotheraputic Agents) are administered via IV, great
care must be taken to prevent the medication from escaping into the tissues surrounding the
injection site, because pain, tissue damage, and necrosis can result. The nurse monitors for
signs of extravasation, such as redness or swelling at the insertion site and a decreased
infusion rate. If extravasation occurs, the registered nurse needs to be notified; he or she will
then contact the health care provider.

13. The client with squamous cell carcinoma of the larynx is receiving bleomycin
intravenously. The nurse caring for the client anticipates that which diagnostic study will be
prescribed?
A. Echocardiography
B. Electrocardiography
C. Cervical radiography
D. Pulmonary function studies
Answer: D. Pulmonary function studies
Rationale:
Bleomycin is an antineoplastic medication (Chemotheraputic Agents) that can cause
interstitial pneumonitis, which can progress to pulmonary fibrosis. Pulmonary function
studies along with hematological, hepatic, and renal function tests need to be monitored. The
nurse needs to monitor lung sounds for dyspnea and crackles, which indicate pulmonary
toxicity. The medication needs to be discontinued immediately if pulmonary toxicity occurs.
Options 1, 2, and 3 are unrelated to the specific use of this medication.
14. The client with acute myelocytic leukemia is being treated with busulfan (Myleran).
Which laboratory value would the nurse specifically monitor during treatment with this
medication?
A. Clotting time
B. Uric acid level
C. Potassium level
D. Blood glucose level
Answer: B. Uric acid level
Rationale:
Busulfan (Myleran) can cause an increase in the uric acid level. Hyperuricemia can produce
uric acid nephropathy, renal stones, and acute renal failure. Options 1, 3, and 4 are not
specifically related to this medication.
15. The client with small cell lung cancer is being treated with etoposide (VePesid). The nurse
who is assisting in caring for the client during its administration understands that which side
effect is specifically associated with this medication?
A. Alopecia

B. Chest pain
C. Pulmonary fibrosis
D. Orthostatic hypotension
Answer: D. Orthostatic hypotension
Rationale:
A side effect specific to etoposide is orthostatic hypotension. The client's blood pressure is
monitored during the infusion. Hair loss occurs with nearly all the antineoplastic medications.
Chest pain and pulmonary fibrosis are unrelated to this medication.
16. The clinic nurse is reviewing a teaching plan for the client receiving an antineoplastic
medication. When implementing the plan, the nurse tells the client:
A. To take aspirin (acetylsalicylic acid) as needed for headache
B. Drink beverages containing alcohol in moderate amounts each evening
C. Consult with health care providers (HCPs) before receiving immunizations
D. That it is not necessary to consult HCPs before receiving a flu vaccine at the local health
fair
Answer: C. Consult with health care providers (HCPs) before receiving immunizations
Rationale:
Because antineoplastic medications lower the resistance of the body, clients must be informed
not to receive immunizations without a HCP's approval. Clients also need to avoid contact
with individuals who have recently received a live virus vaccine. Clients need to avoid
aspirin and aspirin-containing products to minimize the risk of bleeding, and they need to
avoid alcohol to minimize the risk of toxicity and side effects.
17. The client with ovarian cancer is being treated with vincristine (Oncovin). The nurse
monitors the client, knowing that which of the following indicates a side effect specific to this
medication?
A. Diarrhea
B. Hair loss
C. Chest pain
D. Numbness and tingling in the fingers and toes
Answer: D. Numbness and tingling in the fingers and toes
Rationale:

A side effect specific to vincristine is peripheral neuropathy, which occurs in almost every
client. Peripheral neuropathy can be manifested as numbness and tingling in the fingers and
toes. Depression of the Achilles tendon reflex may be the first clinical sign indicating
peripheral neuropathy. Constipation rather than diarrhea is most likely to occur with this
medication, although diarrhea may occur occasionally. Hair loss occurs with nearly all the
antineoplastic medications. Chest pain is unrelated to this medication.
18. The nurse is reviewing the history and physical examination of a client who will be
receiving asparaginase (Elspar), an antineoplastic agent. The nurse consults with the
registered nurse regarding the administration of the medication if which of the following is
documented in the client's history?
A. Pancreatitis
B. Diabetes mellitus
C. Myocardial infarction
D. Chronic obstructive pulmonary disease
Answer: A. Pancreatitis
Rationale:
Asparaginase (Elspar) is contraindicated if hypersensitivity exists, in pancreatitis, or if the
client has a history of pancreatitis. The medication impairs pancreatic function and pancreatic
function tests should be performed before therapy begins and when a week or more has
elapsed between administration of the doses. The client needs to be monitored for signs of
pancreatitis, which include nausea, vomiting, and abdominal pain. The conditions noted in
options 2, 3, and 4 are not contraindicated with this medication.
19. Tamoxifen is prescribed for the client with metastatic breast carcinoma. The nurse
understands that the primary action of this medication is to:
A. Increase DNA and RNA synthesis.
B. Promote the biosynthesis of nucleic acids.
C. Increase estrogen concentration and estrogen response.
D. Compete with estradiol for binding to estrogen in tissues containing high concentrations of
receptors.
Answer: D. Compete with estradiol for binding to estrogen in tissues containing high
concentrations of receptors.
Rationale:

Tamoxifen is an antineoplastic medication that competes with estradiol for binding to
estrogen in tissues containing high concentrations of receptors. Tamoxifen is used to treat
metastatic breast carcinoma in women and men. Tamoxifen is also effective in delaying the
recurrence of cancer following mastectomy. Tamoxifen reduces DNA synthesis and estrogen
response.
20. The client with metastatic breast cancer is receiving tamoxifen. The nurse specifically
monitors which laboratory value while the client is taking this medication?
A. Glucose level
B. Calcium level
C. Potassium level
D. Prothrombin time
Answer: B. Calcium level
Rationale:
Tamoxifen may increase calcium, cholesterol, and triglyceride levels. Before the initiation of
therapy, a complete blood count, platelet count, and serum calcium levels should be assessed.
These blood levels, along with cholesterol and triglyceride levels, should be monitored
periodically during therapy. The nurse should assess for hypercalcemia while the client is
taking this medication. Signs of hypercalcemia include increased urine volume, excessive
thirst, nausea, vomiting, constipation, hypotonicity of muscles, and deep bone and flank pain.
21. A nurse is assisting with caring for a client with cancer who is receiving cisplatin. Select
the adverse effects that the nurse monitors for that are associated with this medication. Select
all that apply.
A. Tinnitus
B. Ototoxicity
C. Hyperkalemia
D. Hypercalcemia
E. Nephrotoxicity
F. Hypomagnesemia
Answer: A. Tinnitus
B. Ototoxicity
E. Nephrotoxicity
F. Hypomagnesemia

Rationale:
Cisplatin is an alkylating medication. Alkylating medications are cell cycle phasenonspecific medications that affect the synthesis of DNA by causing the cross-linking of
DNA to inhibit cell reproduction. Cisplatin may cause ototoxicity, tinnitus, hypokalemia,
hypocalcemia, hypomagnesemia, and nephrotoxicity. Amifostine (Ethyol) may be
administered before cisplatin to reduce the potential for renal toxicity.
22. A nurse is caring for a client after thyroidectomy and notes that calcium gluconate is
prescribed for the client. The nurse determines that this medication has been prescribed to:
A. Treat thyroid storm.
B. Prevent cardiac irritability.
C. Treat hypocalcemic tetany.
D. Stimulate the release of parathyroid hormone.
Answer: C. Treat hypocalcemic tetany.
Rationale:
Hypocalcemia can develop after thyroidectomy if the parathyroid glands are accidentally
removed or injured during surgery. Manifestations develop 1 to 7 days after surgery. If the
client develops numbness and tingling around the mouth, fingertips, or toes or muscle spasms
or twitching, the health care provider is notified immediately. Calcium gluconate should be
kept at the bedside.
23. A client who has been newly diagnosed with diabetes mellitus has been stabilized with
daily insulin injections. Which information should the nurse teach when carrying out plans
for discharge?
A. Keep insulin vials refrigerated at all times.
B. Rotate the insulin injection sites systematically.
C. Increase the amount of insulin before unusual exercise.
D. Monitor the urine acetone level to determine the insulin dosage.
Answer: B. Rotate the insulin injection sites systematically.
Rationale:
Insulin dosages should not be adjusted or increased before unusual exercise. If acetone is
found in the urine, it may possibly indicate the need for additional insulin. To minimize the
discomfort associated with insulin injections, the insulin should be administered at room
temperature. Injection sites should be systematically rotated from one area to another. The

client should be instructed to give injections in one area, about 1 inch apart, until the whole
area has been used and then to change to another site. This prevents dramatic changes in daily
insulin absorption.
24. A nurse is reinforcing teaching for a client regarding how to mix regular insulin and NPH
insulin in the same syringe. Which of the following actions, if performed by the client,
indicates the need for further teaching?
A. Withdraws the NPH insulin first
B. Withdraws the regular insulin first
C. Injects air into NPH insulin vial first
D. Injects an amount of air equal to the desired dose of insulin into the vial
Answer: A. Withdraws the NPH insulin first
Rationale:
When preparing a mixture of regular insulin with another insulin preparation, the regular
insulin is drawn into the syringe first. This sequence will avoid contaminating the vial of
regular insulin with insulin of another type. Options 2, 3, and 4 identify the correct actions for
preparing NPH and regular insulin.
25. A home care nurse visits a client recently diagnosed with diabetes mellitus who is taking
Humulin NPH insulin daily. The client asks the nurse how to store the unopened vials of
insulin. The nurse tells the client to:
A. Freeze the insulin.
B. Refrigerate the insulin.
C. Store the insulin in a dark, dry place
D. Keep the insulin at room temperature.
Answer: B. Refrigerate the insulin.
Rationale:
Insulin in unopened vials should be stored under refrigeration until needed. Vials should not
be frozen. When stored unopened under refrigeration, insulin can be used up to the expiration
date on the vial. Options 1, 3, and 4 are incorrect.
26. Glimepiride (Amaryl) is prescribed for a client with diabetes mellitus. A nurse reinforces
instructions for the client and tells the client to avoid which of the following while taking this
medication?

A. Alcohol
B. Organ meats
C. Whole-grain cereals
D. Carbonated beverages
Answer: A. Alcohol
Rationale:
When alcohol is combined with glimepiride (Amaryl), a disulfiram-like reaction may occur.
This syndrome includes flushing, palpitations, and nausea. Alcohol can also potentiate the
hypoglycemic effects of the medication. Clients need to be instructed to avoid alcohol
consumption while taking this medication. The items in options 2, 3, and 4 do not need to be
avoided.
27. Sildenafil (Viagra) is prescribed to treat a client with erectile dysfunction. A nurse reviews
the client's medical record and would question the prescription if which of the following is
noted in the client's history?
A. Neuralgia
B. Insomnia
C. Use of nitroglycerin
D. Use of multivitamins
Answer: C. Use of nitroglycerin
Rationale:
Sildenafil (Viagra) enhances the vasodilating effect of nitric oxide in the corpus cavernosum
of the penis, thus sustaining an erection. Because of the effect of the medication, it is
contraindicated with concurrent use of organic nitrates and nitroglycerin. Sildenafil is not
contraindicated with the use of vitamins. Neuralgia and insomnia are side effects of the
medication.
28. The health care provider (HCP) prescribes exenatide (Byetta) for a client with type 1
diabetes mellitus who takes insulin. The nurse knows that which of the following is the
appropriate intervention?
A. The medication is administered within 60 minutes before the morning and evening meal.
B. The medication is withheld and the HCP is called to question the prescription for the
client.

C. The client is monitored for gastrointestinal side effects after administration of the
medication.
D. The insulin is withdrawn from the Penlet into an insulin syringe to prepare for
administration.
Answer: B. The medication is withheld and the HCP is called to question the prescription for
the client.
Rationale:
Exenatide (Byetta) is an incretin mimetic used for type 2 diabetes mellitus only. It is not
recommended for clients taking insulin. Hence, the nurse should hold the medication and
question the HCP regarding this prescription. Although options 1 and 3 are correct statements
about the medication, in this situation the medication should not be administered. The
medication is packaged in prefilled pens ready for injection without the need for drawing it
up into another syringe.
29. A client is taking Humulin NPH insulin daily every morning. The nurse reinforces
instructions for the client and tells the client that the most likely time for a hypoglycemic
reaction to occur is:
A. 2 to 4 hours after administration
B. 4 to 12 hours after administration
C. 16 to 18 hours after administration
D. 18 to 24 hours after administration
Answer: B. 4 to 12 hours after administration
Rationale:
Humulin NPH is an intermediate-acting insulin. The onset of action is 1. 5 hours, it peaks in 4
to 12 hours, and its duration of action is 24 hours. Hypoglycemic reactions most likely occur
during peak time.
30. A client with diabetes mellitus visits a health care clinic. The client's diabetes mellitus
previously had been well controlled with glyburide (DiaBeta) daily, but recently the fasting
blood glucose level has been 180 to 200 mg/dL. Which medication, if added to the client's
regimen, may have contributed to the hyperglycemia?
A. Prednisone
B. Phenelzine (Nardil)
C. Atenolol (Tenormin)

D. Allopurinol (Zyloprim)
Answer: A. Prednisone
Rationale:
Prednisone may decrease the effect of oral hypoglycemics, insulin, diuretics, and potassium
supplements. Option 2, a monoamine oxidase inhibitor, and option 3, a β-blocker, have their
own intrinsic hypoglycemic activity. Option 4 decreases urinary excretion of sulfonylurea
agents, causing increased levels of the oral agents, which can lead to hypoglycemia.
31. A community health nurse visits a client at home. Prednisone 10 mg orally daily has been
prescribed for the client and the nurse reinforces teaching for the client about the medication.
Which statement, if made by the client, indicates that further teaching is necessary?
A. "I can take aspirin or my antihistamine if I need it."
B. "I need to take the medication every day at the same time."
C. "I need to avoid coffee, tea, cola, and chocolate in my diet."
D. "If I gain more than 5 pounds a week, I will call my doctor."
Answer: A. "I can take aspirin or my antihistamine if I need it."
Rationale:
Aspirin and other over-the-counter medications should not be taken unless the client consults
with the health care provider (HCP). The client needs to take the medication at the same time
every day and should be instructed not to stop the medication. A slight weight gain as a result
of an improved appetite is expected, but after the dosage is stabilized, a weight gain of 5 lb or
more weekly should be reported to the HCP. Caffeine-containing foods and fluids need to be
avoided because they may contribute to steroid-ulcer development.
32. Desmopressin acetate (DDAVP) is prescribed for the treatment of diabetes insipidus. The
nurse monitors the client after medication administration for which therapeutic response?
A. Decreased urinary output
B. Decreased blood pressure
C. Decreased peripheral edema
D. Decreased blood glucose level
Answer: A. Decreased urinary output
Rationale:
Desmopressin promotes renal conservation of water. The hormone carries out this action by
acting on the collecting ducts of the kidney to increase their permeability to water, which

results in increased water reabsorption. The therapeutic effect of this medication would be
manifested by a decreased urine output. Options 2, 3, and 4 are unrelated to the effects of this
medication.
33. The home health care nurse is visiting a client who was recently diagnosed with type 2
diabetes mellitus. The client is prescribed repaglinide (Prandin) and metformin (Glucophage)
and asks the nurse to explain these medications. The nurse should reinforce which
instructions to the client? Select all that apply.
A. Diarrhea can occur secondary to the metformin.
B. The repaglinide is not taken if a meal is skipped.
C. The repaglinide is taken 30 minutes before eating.
D. Candy or another simple sugar is carried and used to treat mild hypoglycemia episodes.
E. Metformin increases hepatic glucose production to prevent hypoglycemia associated with
repaglinide.
F. Muscle pain is an expected side effect of metformin and may be treated with
acetaminophen (Tylenol).
Answer: A. Diarrhea can occur secondary to the metformin.
B. The repaglinide is not taken if a meal is skipped.
C. The repaglinide is taken 30 minutes before eating.
D. Candy or another simple sugar is carried and used to treat mild hypoglycemia episodes.
Rationale:
Repaglinide is a rapid-acting oral hypoglycemic agent that stimulates pancreatic insulin
secretion that should be taken before meals, and that should be withheld if the client does not
eat. Hypoglycemia is a side effect of repaglinide and the client should always be prepared by
carrying a simple sugar with her or him at all times. Metformin is an oral hypoglycemic given
in combination with repaglinide and works by decreasing hepatic glucose production. A
common side effect of metformin is diarrhea. Muscle pain may occur as an adverse effect
from metformin but it might signify a more serious condition that warrants health care
provider notification, not the use of acetaminophen.
34. A client with Crohn's disease is scheduled to receive an infusion of infliximab
(Remicade). The nurse assisting in caring for the client should take which action to monitor
the effectiveness of treatment?
A. Monitoring the leukocyte count for 2 days after the infusion

B. Checking the frequency and consistency of bowel movements
C. Checking serum liver enzyme levels before and after the infusion
D. Carrying out a Hematest on gastric fluids after the infusion is completed
Answer: C. Checking serum liver enzyme levels before and after the infusion
Rationale:
Checking the frequency and consistency of bowel movements The principal manifestations of
Crohn's disease are diarrhea and abdominal pain. Infliximab (Remicade) is an
immunomodulator that reduces the degree of inflammation in the colon, thereby reducing the
diarrhea. Options 1, 3, and 4 are unrelated to this medication.
35. The client has a PRN prescription for loperamide hydrochloride (Imodium). The nurse
understands that this medication is used for which condition?
A. Constipation
B. Abdominal pain
C. An episode of diarrhea
D. Hematest-positive nasogastric tube drainage
Answer: C. An episode of diarrhea
Rationale:
Loperamide is an antidiarrheal agent. It is used to manage acute and also chronic diarrhea in
conditions such as inflammatory bowel disease. Loperamide also can be used to reduce the
volume of drainage from an ileostomy. It is not used for the conditions in options 1, 2, and 4.
36. The client has a PRN prescription for ondansetron (Zofran). For which condition should
this medication be administered to the postoperative client?
A. Paralytic ileus
B. Incisional pain
C. Urinary retention
D. Nausea and vomiting
Answer: D. Nausea and vomiting
Rationale:
Ondansetron is an antiemetic used to treat postoperative nausea and vomiting, as well as
nausea and vomiting associated with chemotherapy. The other options are incorrect.

37. The client has begun medication therapy with pancrelipase (Pancrease MT). The nurse
evaluates that the medication is having the optimal intended benefit if which effect is
observed?
A. Weight loss
B. Relief of heartburn
C. Reduction of steatorrhea
D. Absence of abdominal pain
Answer: C. Reduction of steatorrhea
Rationale:
Pancrelipase (Pancrease MT) is a pancreatic enzyme used in clients with pancreatitis as a
digestive aid. The medication should reduce the amount of fatty stools (steatorrhea). Another
intended effect could be improved nutritional status. It is not used to treat abdominal pain or
heartburn. Its use could result in weight gain but should not result in weight loss if it is aiding
in digestion.
38. An older client recently has been taking cimetidine (Tagamet). The nurse monitors the
client for which most frequent central nervous system side effect of this medication?
A. Tremors
B. Dizziness
C. Confusion
D. Hallucinations
Answer: C. Confusion
Rationale:
Cimetidine is a histamine 2 (H2)-receptor antagonist. Older clients are especially susceptible
to central nervous system side effects of cimetidine. The most frequent of these is confusion.
Less common central nervous system side effects include headache, dizziness, drowsiness,
and hallucinations.
39. The client with a gastric ulcer has a prescription for sucralfate (Carafate), 1 g by mouth
four times daily. The nurse schedules the medication for which times?
A. With meals and at bedtime
B. Every 6 hours around the clock
C. One hour after meals and at bedtime
D. One hour before meals and at bedtime

Answer: D. One hour before meals and at bedtime
Rationale:
Sucralfate is a gastric protectant. The medication should be scheduled for administration 1
hour before meals and at bedtime. The medication is timed to allow it to form a protective
coating over the ulcer before food intake stimulates gastric acid production and mechanical
irritation. The other options are incorrect.
40. The client who chronically uses nonsteroidal anti- inflammatory drugs has been taking
misoprostol (Cytotec). The nurse determines that the medication is having the intended
therapeutic effect if which of the following is noted?
A. Resolved diarrhea
B. Relief of epigastric pain
C. Decreased platelet count
D. Decreased white blood cell count
Answer: B. Relief of epigastric pain
Rationale:
The client who chronically uses nonsteroidal anti- inflammatory drugs (NSAIDs) is prone to
gastric mucosal injury. Misoprostol is a gastric protectant and is given specifically to prevent
this occurrence. Diarrhea can be a side effect of the medication, but is not an intended effect.
Options 3 and 4 are incorrect.
41. The client has been taking omeprazole (Prilosec) for 4 weeks. The ambulatory care nurse
evaluates that the client is receiving optimal intended effect of the medication if the client
reports the absence of which symptom?
A. Diarrhea
B. Heartburn
C. Flatulence
D. Constipation
Answer: B. Heartburn
Rationale:
Omeprazole is a proton pump inhibitor classified as an antiulcer agent. The intended effect of
the medication is relief of pain from gastric irritation, often called heartburn by clients.
Omeprazole is not used to treat the conditions identified in options 1, 3, and 4.

42. A client with a peptic ulcer is diagnosed with a Helicobacter pylori infection. The nurse is
reinforcing teaching for the client about the medications prescribed, including clarithromycin
(Biaxin), esomeprazole (Nexium), and amoxicillin (Amoxil). Which statement by the client
indicates the best understanding of the medication regimen?
A. "My ulcer will heal because these medications will kill the bacteria."
B. "These medications are only taken when I have pain from my ulcer."
C. "The medications will kill the bacteria and stop the acid production."
D. "These medications will coat the ulcer and decrease the acid production in my stomach."
Answer: C. "The medications will kill the bacteria and stop the acid production."
Rationale:
Triple therapy for Helicobacter pylori infection usually includes two antibacterial drugs and a
proton pump inhibitor. Clarithromycin and amoxicillin are antibacterials. Esomeprazole is a
proton pump inhibitor. These medications will kill the bacteria and decrease acid production.
43. A histamine (H2)-receptor antagonist will be prescribed for a client. The nurse
understands that which medications are H2-receptor antagonists? Select all that apply.
A. Nizatidine (Axid)
B. Ranitidine (Zantac)
C. Famotidine (Pepcid)
D. Cimetidine (Tagamet)
E. Esomeprazole (Nexium)
F. Lansoprazole (Prevacid)
Answer: A. Nizatidine (Axid)
C. Ranitidine (Zantac)
C. Famotidine (Pepcid)
D. Cimetidine (Tagamet)
Rationale:
H2-receptor antagonists suppress secretion of gastric acid, alleviate symptoms of heartburn,
and assist in preventing complications of peptic ulcer disease. These medications also
suppress gastric acid secretions and are used in active ulcer disease, erosive esophagitis, and
pathological hypersecretory conditions. The other medications listed are proton pump
inhibitors. H2-receptor antagonists medication names end with -dine. Proton pump inhibitors
medication names end with -zole.

44. A client is receiving acetylcysteine (Mucomyst), 20% solution diluted in 0.9% normal
saline by nebulizer. The nurse should have which item available for possible use after giving
this medication?
A. Ambu bag
B. Intubation tray
C. Nasogastric tube
D. Suction equipment
Answer: D. Suction equipment
Rationale:
Acetylcysteine can be given orally or by nasogastric tube to treat acetaminophen overdose, or
it may be given by inhalation for use as a mucolytic. The nurse administering this medication
as a mucolytic should have suction equipment available in case the client cannot manage to
clear the increased volume of liquefied secretions.
45. A client has a prescription to take guaifenesin (Humibid) every 4 hours, as needed. The
nurse determines that the client understands the most effective use of this medication if the
client states that he or she will:
A. Watch for irritability as a side effect.
B. Take the tablet with a full glass of water.
C. Take an extra dose if the cough is accompanied by fever.
D. Crush the sustained-release tablet if immediate relief is needed.
Answer: B. Take the tablet with a full glass of water.
Rationale:
Guaifenesin is an expectorant. It should be taken with a full glass of water to decrease
viscosity of secretions. Sustained-release preparations should not be broken open, crushed, or
chewed. The medication may occasionally cause dizziness, headache, or drowsiness as side
effects. The client should contact the health care provider if the cough lasts longer than 1
week or is accompanied by fever, rash, sore throat, or persistent headache.
46. A postoperative client has received a dose of naloxone hydrochloride for respiratory
depression shortly after transfer to the nursing unit from the postanesthesia care unit. After
administration of the medication, the nurse checks the client for:
A. Pupillary changes
B. Scattered lung wheezes

C. Sudden increase in pain
D. Sudden episodes of diarrhea
Answer: C. Sudden increase in pain
Rationale:
Naloxone hydrochloride is an antidote to opioids and may also be given to the postoperative
client to treat respiratory depression. When given to the postoperative client for respiratory
depression, it may also reverse the effects of analgesics. Therefore, the nurse must check the
client for a sudden increase in the level of pain experienced. Options 1, 2, and 4 are not
associated with this medication.
47. A client has been taking isoniazid (INH) for 2 months. The client complains to a nurse
about numbness, paresthesias, and tingling in the extremities. The nurse interprets that the
client is experiencing:
A. Hypercalcemia
B. Peripheral neuritis
C. Small blood vessel spasm
D. Impaired peripheral circulation
Answer: B. Peripheral neuritis
Rationale:
A common side effect of the TB drug INH is peripheral neuritis. This is manifested by
numbness, tingling, and paresthesias in the extremities. This side effect can be minimized by
pyridoxine (vitamin B6) intake. Options 1, 3, and 4 are incorrect.
48. A client is to begin a 6- month course of therapy with isoniazid (INH). A nurse plans to
teach the client to:
A. Drink alcohol in small amounts only.
B. Report yellow eyes or skin immediately.
C. Increase intake of Swiss or aged cheeses.
D. Avoid vitamin supplements during therapy.
Answer: B. Report yellow eyes or skin immediately
Rationale:
INH is hepatotoxic, and therefore the client is taught to report signs and symptoms of
hepatitis immediately (which include yellow skin and sclera). For the same reason, alcohol
should be avoided during therapy. The client should avoid intake of Swiss cheese, fish such

as tuna, and foods containing tyramine because they may cause a reaction characterized by
redness and itching of the skin, flushing, sweating, tachycardia, headache, or light
headedness. The client can avoid developing peripheral neuritis by increasing the intake of
pyridoxine (vitamin B6) during the course of INH therapy for TB.
49. A client has been started on long-term therapy with rifampin (Rifadin). A nurse teaches
the client that the medication:
A. Should always be taken with food or antacids
B. Should be double-dosed if one dose is forgotten
C. Causes orange discoloration of sweat, tears, urine, and feces
D. May be discontinued independently if symptoms are gone in 3 months
Answer: C. Causes orange discoloration of sweat, tears, urine, and feces
Rationale:
Rifampin should be taken exactly as directed as part of TB therapy. Doses should not be
doubled or skipped. The client should not stop therapy until directed to do so by a health care
provider. The medication should be administered on an empty stomach unless it causes
gastrointestinal upset, and then it may be taken with food. Antacids, if prescribed, should be
taken at least 1 hour before the medication. Rifampin causes orange-red discoloration of body
secretions and will permanently stain soft contact lenses.
50. A nurse has given a client taking ethambutol (Myambutol) information about the
medication. The nurse determines that the client understands the instructions if the client
states that he or she will immediately report:
A. Impaired sense of hearing
B. Problems with visual acuity
C. Gastrointestinal (GI) side effects
D. Orange-red discoloration of body secretions
Answer: B. Problems with visual acuity
Rationale:
Ethambutol causes optic neuritis, which decreases visual acuity and the ability to discriminate
between the colors red and green. This poses a potential safety hazard when a client is driving
a motor vehicle. The client is taught to report this symptom immediately. The client is also
taught to take the medication with food if GI upset occurs. Impaired hearing results from

antitubercular therapy with streptomycin. Orange- red discoloration of secretions occurs with
rifampin (Rifadin).
51. Cycloserine (Seromycin) is added to the medication regimen for a client with
tuberculosis. Which of the following would the nurse include in the client- teaching plan
regarding this medication?
A. To take the medication before meals
B. To return to the clinic weekly for serum drug-level testing
C. It is not necessary to call the health care provider (HCP) if a skin rash occurs.
D. It is not necessary to restrict alcohol intake with this medication.
Answer: B. To return to the clinic weekly for serum drug-level testing
Rationale:
Cycloserine (Seromycin) is an antitubercular medication that requires weekly serum drug
level determinations to monitor for the potential of neurotoxicity. Serum drug levels lower
than 30 mcg/mL reduce the incidence of neurotoxicity. The medication must be taken after
meals to prevent gastrointestinal irritation. The client must be instructed to notify the HCP if
a skin rash or signs of central nervous system toxicity are noted. Alcohol must be avoided
because it increases the risk of seizure activity.
52. A client with tuberculosis is being started on antituberculosis therapy with isoniazid
(INH). Before giving the client the first dose, a nurse ensures that which of the following
baseline studies has been completed?
A. Electrolyte levels
B. Coagulation times
C. Liver enzyme levels
D. Serum creatinine level
Answer: C. Liver enzyme levels
Rationale:
INH therapy can cause an elevation of hepatic enzyme levels and hepatitis. Therefore, liver
enzyme levels are monitored when therapy is initiated and during the first 3 months of
therapy. They may be monitored longer in the client who is greater than age 50 or abuses
alcohol.

53. Rifabutin (Mycobutin) is prescribed for a client with active Mycobacterium avium
complex (MAC) disease and tuberculosis. The nurse monitors for which side effects of the
medication? Select all that apply.
A. Signs of hepatitis
B. Flu-like syndrome
C. Low neutrophil count
D. Vitamin B6 deficiency
E. Ocular pain or blurred vision
F. Tingling and numbness of the fingers
Answer: A. Signs of hepatitis
B. Flu-like syndrome
C. Low neutrophil count
E. Ocular pain or blurred vision
Rationale:
Rifabutin (Mycobutin) may be prescribed for a client with active MAC disease and
tuberculosis. It inhibits mycobacterial DNA-dependent RNA polymerase and suppresses
protein synthesis. Side effects include rash, gastrointestinal disturbances, neutropenia (low
neutrophil count), red-orange body secretions, uveitis (blurred vision and eye pain), myositis,
arthralgia, hepatitis, chest pain with dyspnea, and flu- like syndrome. Vitamin B6 deficiency
and numbness and tingling in the extremities are associated with the use of isoniazid (INH).
Ethambutol (Myambutol) also causes peripheral neuritis.
54. A nurse reinforces discharge instructions to a postoperative client who is taking warfarin
sodium (Coumadin). Which statement, if made by the client, reflects the need for further
teaching?
A. "I will take my pills every day at the same time."
B. "I will be certain to avoid alcohol consumption."
C. "I have already called my family to pick up a Medic-Alert bracelet."
D. "I will take Ecotrin (enteric-coated aspirin) for my headaches because it is coated."
Answer: D. "I will take Ecotrin (enteric-coated aspirin) for my headaches because it is
coated."
Rationale:
Ecotrin is an aspirin-containing product and should be avoided. Alcohol consumption should
be avoided by a client taking warfarin sodium. Taking prescribed medication at the same time

each day increases client compliance. The Medic-Alert bracelet provides health care
personnel emergency information.
55. A client who is receiving digoxin (Lanoxin) daily has a serum potassium level of 3. 0
mEq/L and is complaining of anorexia. A health care provider prescribes a digoxin level to
rule out digoxin toxicity. A nurse checks the results, knowing that which of the following is
the therapeutic serum level (range) for digoxin?
A. 3 to 5 ng/mL
B. 0.5 to 2 ng/mL
C. 3. 1. 2 to 2. 8 ng/mL
D. 3. 5 to 5. 5ng/m:
Answer: B. 0.5 to 2 ng/mL
Rationale:
Therapeutic levels for digoxin range from 0.5 to 2 ng/mL. Therefore, options 1, 3, and 4 are
incorrect.
56. Heparin sodium is prescribed for the client. The nurse expects that the health care
provider will prescribe which of the following to monitor for a therapeutic effect of the
medication?
A. Hematocrit level
B. Hemoglobin level
C. Prothrombin time (PT)
D. Activated partial thromboplastin time (aPTT)
Answer: D. Activated partial thromboplastin time (aPTT)
Rationale:
The PT will assess for the therapeutic effect of warfarin sodium (Coumadin) and the aPTT
will assess the therapeutic effect of heparin sodium. Heparin sodium doses are determined
based on these laboratory results. The hemoglobin and hematocrit values assess red blood cell
concentrations
57. A nurse is monitoring a client who is taking propranolol (Inderal LA). Which data
collection finding would indicate a potential serious complication associated with
propranolol?
A. The development of complaints of insomnia

B. The development of audible expiratory wheezes
C. A baseline blood pressure of 150/80 mm Hg followed by a blood pressure of 138/72 mm
Hg after two doses of the medication
D. A baseline resting heart rate of 88 beats/min followed by a resting heart rate of 72
beats/min after two doses of the medication
Answer: B. The development of audible expiratory wheezes
Rationale:
Audible expiratory wheezes may indicate a serious adverse reaction, bronchospasm. βBlockers may induce this reaction, particularly in clients with chronic obstructive pulmonary
disease or asthma. Normal decreases in blood pressure and heart rate are expected. Insomnia
is a frequent mild side effect and should be monitored.
58. Isosorbide mononitrate (Imdur) is prescribed for a client with angina pectoris. The client
tells the nurse that the medication is causing a chronic headache. The nurse appropriately
suggests that the client:
A. Cut the dose in half.
B. Discontinue the medication.
C. Take the medication with food.
D. Contact the health care provider (HCP).
Answer: C. Take the medication with food.
Rationale:
Isosorbide mononitrate is an antianginal medication. Headache is a frequent side effect of
isosorbide mononitrate and usually disappears during continued therapy. If a headache occurs
during therapy, the client should be instructed to take the medication with food or meals. It is
not necessary to contact the HCP unless the headaches persist with therapy. It is not
appropriate to instruct the client to discontinue therapy or adjust the dosages.
59. A client is diagnosed with an acute myocardialinfarction and is receiving tissue
plasminogen activator, alteplase (Activase, tPA). Which action is a priority nursing
intervention?
A. Monitor for renal failure.
B. Monitor psychosocial status.
C. Monitor for signs of bleeding.
D. Have heparin sodium available.

Answer: C. Monitor for signs of bleeding.
Rationale:
Tissue plasminogen activator is a thrombolytic. Hemorrhage is a complication of any type of
thrombolytic medication. The client is monitored for bleeding. Monitoring for renal failure
and monitoring the client's psychosocial status are important but are not the most critical
interventions. Heparin is given after thrombolytic therapy, but the question is not asking
about follow-up medications.
60. A nurse is planning to administer hydrochlorothiazide (HydroDIURIL) to a client. The
nurse understands that which of the following are concerns related to the administration of
this medication?
A. Hypouricemia, hyperkalemia
B. Increased risk of osteoporosis
C. Hypokalemia, hyperglycemia, sulfa allergy
D. Hyperkalemia, hypoglycemia, penicillin allergy
Answer: C. Hypokalemia, hyperglycemia, sulfa allergy
Rationale:
Thiazide diuretics such as hydrochlorothiazide are sulfa-based medications, and a client with
a sulfa allergy is at risk for an allergic reaction. Also, clients are at risk for hypokalemia,
hyperglycemia, hypercalcemia, hyperlipidemia, and hyperuricemia.
61. A home health care nurse is visiting a client with elevated triglyceride levels and a serum
cholesterol level of 398 mg/dL. The client is taking cholestyramine (Questran). Which of the
following statements, if made by the client, indicates the need for further education?
A. "Constipation and bloating might be a problem."
B. "I'll continue to watch my diet and reduce my fats."
C. "Walking a mile each day will help the whole process."
D. "I'll continue my nicotinic acid from the health food store."
Answer: D. "I'll continue my nicotinic acid from the health food store."
Rationale:
Nicotinic acid, even an over-the-counter form, should be avoided because it may lead to liver
abnormalities. All lipid-lowering medications also can cause liver abnormalities, so a
combination of nicotinic acid and cholestyramine resin is to be avoided. Constipation and

bloating are the two most common side effects. Walking and the reduction of fats in the diet
are therapeutic measures to reduce cholesterol and triglyceride levels.
62. A client is on nicotinic acid (niacin) for hyperlipidemia and the nurse provides
instructions to the client about the medication. Which statement by the client would indicate
an understanding of the instructions?
A. "It is not necessary to avoid the use of alcohol."
B. "The medication should be taken with meals to decrease flushing."
C. "Clay-colored stools are a common side effect and should not be of concern."
D. "Ibuprofen (Motrin) taken 30 minutes before the nicotinic acid should decrease the
flushing."
Answer: D. "Ibuprofen (Motrin) taken 30 minutes before the nicotinic acid should decrease
the flushing."
Rationale:
Flushing is a side effect of this medication. Aspirin or a nonsteroidal anti- inflammatory drug
can be taken 30 minutes before taking the medication to decrease flushing. Alcohol
consumption needs to be avoided because it will enhance this side effect. The medication
should be taken with meals, this will decrease gastrointestinal upset. Taking the medication
with meals has no effect on the flushing. Clay-colored stools are a sign of hepatic dysfunction
and should be immediately reported to the health care provider (HCP).
63. A client with coronary artery disease complains of substernal chest pain. After checking
the client's heart rate and blood pressure, a nurse administers nitroglycerin, 0.4 mg,
sublingually. After 5 minutes, the client states, "My chest still hurts." Select the appropriate
actions that the nurse should take. Select all that apply.
A. Call a code blue.
B. Contact the registered nurse.
C. Contact the client's family.
D. Assess the client's pain level.
E. Check the client's blood pressure.
F. Administer a second nitroglycerin, 0.4 mg, sublingually.
Answer: B. Contact the registered nurse.
D. Assess the client's pain level.
E. Check the client's blood pressure.

F. Administer a second nitroglycerin, 0. 4 m g, sublingually.
Rationale:
The usual guideline for administering nitroglycerin tablets for a hospitalized client with chest
pain is to administer one tablet every 5 minutes PRN for chest pain, for a total dose of three
tablets. The registered nurse should be notified of the client's condition, who will then notify
the health care provider as appropriate. Because the client is still complaining of chest pain,
the nurse would administer a second nitroglycerin tablet. The nurse would assess the client's
pain level and check the client's blood pressure before administering each nitroglycerin dose.
There are no data in the question that indicate the need to call a code blue. In addition, it is
not necessary to contact the client's family unless the client has requested this
64. Nalidixic acid (NegGram) is prescribed for a client with a urinary tract infection. On
review of the client's record, the nurse notes that the client is taking warfarin sodium
(Coumadin) daily. Which prescription should the nurse anticipate for this client?
A. Discontinuation of warfarin sodium (Coumadin)
B. A decrease in the warfarin sodium (Coumadin) dosage
C. An increase in the warfarin sodium (Coumadin) dosage
D. A decrease in the usual dose of nalidixic acid (NegGram)
Answer: B. A decrease in the warfarin sodium (Coumadin) dosage
Rationale:
Nalidixic acid can intensify the effects of oral anticoagulants by displacing these agents from
binding sites on plasma protein. When an oral anticoagulant is combined with nalidixic acid,
a decrease in the anticoagulant dosage may be needed.
65. A nurse is reinforcing discharge instructions to a client receiving sulfisoxazole. Which of
the following should be included in the list of instructions?
A. Restrict fluid intake.
B. Maintain a high fluid intake.
C. If the urine turns dark brown, call the health care provider (HCP) immediately.
D. Decrease the dosage when symptoms are improving to prevent an allergic response.
Answer: B. Maintain a high fluid intake.
Rationale:
Each dose of sulfisoxazole should be administered with a full glass of water, and the client
should maintain a high fluid intake. The medication is more soluble in alkaline urine. The

client should not be instructed to taper or discontinue the dose. Some forms of sulfisoxazole
cause urine to turn dark brown or red. This does not indicate the need to notify the HCP.
66. Trimethoprim-sulfamethoxazole (TMP-SMZ) is prescribed for a client. A nurse should
instruct the client to report which symptom if it developed during the course of this
medication therapy?
A. Nausea
B. Diarrhea
C. Headache
D. Sore throat
Answer: D. Sore throat
Rationale:
Clients taking trimethoprim-sulfamethoxazole (TMP-SMZ) should be informed about early
signs of blood disorders that can occur from this medication. These include sore throat, fever,
and pallor, and the client should be instructed to notify the health care provider if these
symptoms occur. The other options do not require health care provider notification.
67. Phenazopyridine hydrochloride (Pyridium) is prescribed for a client for symptomatic
relief of pain resulting from a lower urinary tract infection. The nurse reinforces to the client:
A. To take the medication at bedtime
B. To take the medication before meals
C. To discontinue the medication if a headache occurs
D. That a reddish orange discoloration of the urine may occur
Answer: D. That a reddish orange discoloration of the urine may occur
Rationale:
The nurse should instruct the client that a reddish-orange discoloration of urine may occur.
The nurse also should instruct the client that this discoloration can stain fabric. The
medication should be taken after meals to reduce the possibility of gastrointestinal upset. A
headache is an occasional side effect of the medication and does not warrant discontinuation
of the medication.
68. Bethanechol chloride (Urecholine) is prescribed for a client with urinary retention. Which
disorder would be a contraindication to the administration of this medication?
A. Gastric atony

B. Urinary strictures
C. Neurogenic atony
D. Gastroesophageal reflux
Answer: B. Urinary strictures
Rationale:
Bethanechol chloride (Urecholine) can be harmful to clients with urinary tract obstruction or
weakness of the bladder wall. The medication has the ability to contract the bladder and
thereby increase pressure within the urinary tract. Elevation of pressure within the urinary
tract could rupture the bladder in clients with these conditions.
69. A nurse who is administering bethanechol chloride (Urecholine) is monitoring for acute
toxicity associated with the medication. The nurse checks the client for which sign of
toxicity?
A. Dry skin
B. Dry mouth
C. Bradycardia
D. Signs of dehydration
Answer: C. Bradycardia
Rationale:
Toxicity (overdose) produces manifestations of excessive muscarinic stimulation such as
salivation, sweating, involuntary urination and defecation, bradycardia, and severe
hypotension. Treatment includes supportive measures and the administration of atropine
sulfate subcutaneously or intravenously.
70. Oxybutynin chloride (Ditropan XL) is prescribed for a client with neurogenic bladder.
Which sign would indicate a possible toxic effect related to this medication?
A. Pallor
B. Drowsiness
C. Bradycardia
D. Restlessness
Answer: D. Restlessness
Rationale:
Toxicity (overdosage) of this medication produces central nervous system excitation, such as
nervousness, restlessness, hallucinations, and irritability. Other signs of toxicity include

hypotension or hypertension, confusion, tachycardia, flushed or red face, and signs of
respiratory depression. Drowsiness is a frequent side effect of the medication but does not
indicate overdosage.
71. After kidney transplantation, cyclosporine (Sand immune) is prescribed for a client.
Which laboratory result would indicate an adverse effect from the use of this medication?
A. Decreased creatinine level
B. Decreased hemoglobin level
C. Elevated blood urea nitrogen level
D. Decreased white blood cell count
Answer: C. Elevated blood urea nitrogen level
Rationale:
Nephrotoxicity can occur from the use of cyclosporine (Sandimmune). Nephrotoxicity is
evaluated by monitoring for elevated blood urea nitrogen (BUN) and serum creatinine levels.
Cyclosporine is an immunosuppressant but does not depress the bone marrow.
72. Cinoxacin (Cinobac), a urinary antiseptic, is prescribed for the client. The nurse reviews
the client's medical record and should contact the health care provider (HCP) regarding which
documented finding to verify the prescription? Refer to chart.
A. Renal insufficiency
B. Chest x-ray: normal
C. Blood glucose, 102 mg/dL
D. Folic acid (vitamin B6) 0.5 mg, orally daily
Answer: A. Renal insufficiency
Rationale:
Cinoxacin should be administered with caution in clients with renal impairment. The dosage
should be reduced, and failure to do so could result in accumulation of cinoxacin to toxic
levels. Therefore the nurse would verify the prescription if the client had a documented
history of renal insufficiency. The laboratory and diagnostic test results are normal findings.
Folic acid (vitamin B6) may be prescribed for a client with renal insufficiency to prevent
anemia.
73. A client with myasthenia gravis is suspected of having cholinergic crisis. Which of the
following indicate that this crisis exists?

A. Ataxia
B. Mouth sores
C. Hypotension
D. Hypertension
Answer: D. Hypertension
Rationale:
Cholinergic crisis occurs as a result of an overdose of medication Indications of cholinergic
crisis include gastrointestinal disturbances nausea, vomiting, diarrhea, abdominal cramps,
increased salivation and tearing, miosis, hypertension, sweating, and increased bronchial
secretions.
74. A client with myasthenia gravis is receiving pyridostigmine (Mestinon). The nurse
monitors for signs and symptoms of cholinergic crisis caused by overdose of the medication.
The nurse checks the medication supply to ensure that which medication is available for
administration if a cholinergic crisis occurs?
A. Vitamin K
B. Atropine sulfate
C. Protamine sulfate
D. Acetylcysteine (Mucomyst)
Answer: B. Atropine sulfate
Rationale:
The antidote for cholinergic crisis is atropine sulfate. Vitamin K is the antidote for warfarin
(Coumadin). Protamine sulfate is the antidote for heparin, and acetylcysteine (Mucomyst) is
the antidote for acetaminophen (Tylenol).
75. A client with myasthenia gravis becomes increasingly weak. The health care provider
prepares to identify whether the client is reacting to an overdose of the medication
(cholinergic crisis) or increasing severity of the disease (myasthenic crisis). An injection of
edrophonium (Enlon) is administered. Which of the following indicates that the client is in
cholinergic crisis?
A. No change in the condition
B. Complaints of muscle spasms
C. An improvement of the weakness
D. A temporary worsening of the condition

Answer: D. A temporary worsening of the condition
Rationale:
An edrophonium (Enlon) injection, a cholinergic drug, makes the client in cholinergic crisis
temporarily worse. This is known as a negative test. An improvement of weakness would
occur if the client were experiencing myasthenia gravis. Options 1 and 2 would not occur in
either crisis.
76. Carbidopa-levodopa (Sinemet) is prescribed for a client with Parkinson's disease, and the
nurse monitors the client for adverse reactions to the medication. Which of the following
indicates that the client is experiencing an adverse reaction?
A. Pruritus
B. Tachycardia
C. Hypertension
D. Impaired voluntary movements
Answer: D. Impaired voluntary movements
Rationale:
Dyskinesia and impaired voluntary movement may occur with high levodopa dosages.
Nausea, anorexia, dizziness, orthostatic hypotension, bradycardia, and akinesia (the
temporary muscle weakness that lasts 1 minute to 1 hour, also known as the "on-off
phenomenon") are frequent side effects of the medication.
77. Phenytoin (Dilantin), 100 mg orally three times daily, has been prescribed for a client for
seizure control. The nurse reinforces instructions regarding the medication to the client.
Which statement by the client indicates an understanding of the instructions?
A. "I will use a soft toothbrush to brush my teeth."
B. "It's all right to break the capsules to make it easier for me to swallow them."
C. "If I forget to take my medication, I can wait until the next dose and eliminate that dose."
D. "If my throat becomes sore, it's a normal effect of the medication and it's nothing to be
concerned about."
Answer: A. "I will use a soft toothbrush to brush my teeth."
Rationale:
Phenytoin (Dilantin) is an anticonvulsant. Gingival hyperplasia, bleeding, swelling, and
tenderness of the gums can occur with the use of this medication. The client needs to be
taught good oral hygiene, gum massage, and the need for regular dentist visits. The client

should not skip medication doses, because this could precipitate a seizure. Capsules should
not be chewed or broken and they must be swallowed. The client needs to be instructed to
report a sore throat, fever, glandular swelling, or any skin reaction, because this indicates
hematological toxicity.
78. A client is taking phenytoin (Dilantin) for seizure control and a sample for a serum drug
level is drawn. Which of the following indicates a therapeutic serum drug range?
A. 5 to 10 mcg/mL
B. 10 to 20 mcg/mL
C. 20 to 30 mcg/mL
D. 30 to 40 mcg/mL
Answer: B. 10 to 20 mcg/mL
Rationale:
The therapeutic serum drug level range for phenytoin (Dilantin) is 10 to 20 mcg/mL. * A
helpful hint may be to remember that the theophylline therapeutic range and the
acetaminophen (Tylenol) therapeutic range are the same as the phenytoin (Dilantin)
therapeutic range.*
79. Ibuprofen (Advil) is prescribed for a client. The nurse tells the client to take the
medication:
A. With 8 oz of milk
B. In the morning after arising
C. 60 minutes before breakfast
D. At bedtime on an empty stomach
Answer: A. With 8 oz of milk
Rationale:
Ibuprofen is a nonsteroidal anti-inflammatory drug (NSAID). NSAIDs should be given with
milk or food to prevent gastrointestinal irritation. Options 2, 3, and 4 are incorrect.
80. A nurse is caring for a client who is taking phenytoin (Dilantin) for control of seizures.
During data collection, the nurse notes that the client is taking birth control pills. Which of
the following information should the nurse provide to the client?
A. Pregnancy should be avoided while taking phenytoin (Dilantin).

B. The client may stop taking the phenytoin (Dilantin) if it is causing severe gastrointestinal
effects.
C. The potential for decreased effectiveness of the birth control pills exists while taking
phenytoin (Dilantin).
D. The increased risk of thrombophlebitis exists while taking phenytoin (Dilantin) and birth
control pills together
Answer: C. The potential for decreased effectiveness of the birth control pills exists while
taking phenytoin (Dilantin).
Rationale:
Phenytoin (Dilantin) enhances the rate of estrogen metabolism, which can decrease the
effectiveness of some birth control pills. Options 1, 2, are 4 are not accurate.
81. A client with trigeminal neuralgia is being treated with carbamazepine (Tegretol). Which
laboratory result would indicate that the client is experiencing an adverse reaction to the
medication?
A. Sodium level, 140 mEq/L
B. Uric acid level, 5. 0 mg/dL
C. White blood cell count, 3000 cells/mm3
D. Blood urea nitrogen (BUN) level, 15 mg/dL
Answer: C. White blood cell count, 3000 cells/mm3
Rationale:
Adverse effects of carbamazepine (Tegretol) appear as blood dyscrasias, including aplastic
anemia, agranulocytosis, thrombocytopenia, leukopenia, cardiovascular disturbances,
thrombophlebitis, dysrhythmias, and dermatological effects. Options 1, 2, and 4 identify
normal laboratory values.
82. A client is receiving meperidine hydrochloride (Demerol) for pain. Which of the
following are side effects of this medication. Select all that apply.
A. Diarrhea
B. Tremors
C. Drowsiness
D. Hypotension
E. Urinary frequency
F. Increased respiratory rate

Answer: B. Tremors
C. Drowsiness
D. Hypotension
Rationale:
Meperidine hydrochloride is an opioid analgesic. Side effects include respiratory depression,
drowsiness, hypotension, constipation, urinary retention, nausea, vomiting, and tremors.
83. The client has been on treatment for rheumatoid arthritis for 3 weeks. During the
administration of etanercept (Enbrel), it is most important for the nurse to check:
A. The injection site for itching and edema
B. The white blood cell counts and platelet counts
C. Whether the client is experiencing fatigue and joint pain
D. A metallic taste in the mouth, with a loss of appetite
Answer: B. The white blood cell counts and platelet counts
Rationale:
Infection and pancytopenia are side effects of etanercept (Enbrel). Laboratory studies are
performed before and during drug treatment. The appearance of abnormal white blood cell
counts and abnormal platelet counts can alert the nurse to a potentially life- threatening
infection. Injection site itching is a common occurrence following administration. A metallic
taste with loss of appetite are not common signs of side effects of this medication.
84. Baclofen (Lioresal) is prescribed for the client with multiple sclerosis. The nurse assists in
planning care, knowing that the primary therapeutic effect of this medication is which of the
following?
A. Increased muscle tone
B. Decreased muscle spasms
C. Increased range of motion
D. Decreased local pain and tenderness
Answer: B. Decreased muscle spasms
Rationale:
Baclofen is a skeletal muscle relaxant and central nervous system depressant and acts at the
spinal cord level to decrease the frequency and amplitude of muscle spasms in clients with
spinal cord injuries or diseases and in clients with multiple sclerosis. Options 1, 3, and 4 are
incorrect.

85. A nurse is monitoring a client receiving baclofen (Lioresal) for side effects related to the
medication. Which of the following would indicate that the client is experiencing a side
effect?
A. Polyuria
B. Diarrhea
C. Drowsiness
D. Muscular excitability
Answer: C. Drowsiness
Rationale:
Baclofen is a central nervous system (CNS) depressant and frequently causes drowsiness,
dizziness, weakness, and fatigue. It can also cause nausea, constipation, and urinary retention.
Clients should be warned about the possible reactions. Options 1, 2, and 4 are not side effects.
86. A nurse is reinforcing discharge instructions to a client receiving baclofen (Lioresal).
Which of the following would the nurse include in the instructions?
A. Restrict fluid intake.
B. Avoid the use of alcohol.
C. Stop the medication if diarrhea occurs.
D. Notify the health care provider if fatigue occurs.
Answer: B. Avoid the use of alcohol.
Rationale:
Baclofen is a central nervous system (CNS) depressant. The client should be cautioned
against the use of alcohol and other CNS depressants, because baclofen potentiates the
depressant activity of these agents. Constipation rather than diarrhea is an adverse effect of
baclofen. It is not necessary to restrict fluids, but the client should be warned that urinary
retention can occur. Fatigue is related to a CNS effect that is most intense during the early
phase of therapy and diminishes with continued medication use. It is not necessary that the
client notify the health care provider if fatigue occurs.
87. A client with acute muscle spasms has been taking baclofen (Lioresal). The client calls
the clinic nurse because of continuous feelings of weakness and fatigue and asks the nurse
about discontinuing the medication. The nurse should make which appropriate response to
the client?

A. "You should never stop the medication."
B. "It is best that you taper the dose if you intend to stop the medication."
C. "It is okay to stop the medication if you think that you can tolerate the muscle spasms."
D. "Weakness and fatigue commonly occur and will diminish with continued medication
use."
Answer: D. "Weakness and fatigue commonly occur and will diminish with continued
medication use."
Rationale:
The client should be instructed that symptoms such as drowsiness, weakness, and fatigue are
more intense in the early phase of therapy and diminish with continued medication use. The
client should be instructed never to withdraw or stop the medication abruptly, because abrupt
withdrawal can cause visual hallucinations, paranoid ideation, and seizures. It is best for the
nurse to inform the client that these symptoms will subside and encourage the client to
continue the use of the medication.
88. Dantrolene sodium (Dantrium) is prescribed for a client experiencing flexor spasms, and
the client asks the nurse about the action of the medication. The nurse responds, knowing that
the therapeutic action of this medication is which of the following?
A. Depresses spinal reflexes
B. Acts directly on the skeletal muscle to relieve spasticity
C. Acts within the spinal cord to suppress hyperactive reflexes
D. Acts on the central nervous system (CNS) to suppress spasms
Answer: B. Acts directly on the skeletal muscle to relieve Spasticity
Rationale:
Dantrium acts directly on skeletal muscle to relieve muscle spasticity. The primary action is
the suppression of calcium release from the sarcoplasmic reticulum. This in turn decreases
the ability of the skeletal muscle to contract. *Options 1, 3, and 4 are all comparable or alike
in that they address CNS suppression and the depression of reflexes. Therefore, eliminate
these options.*
89. A nurse is reviewing the laboratory studies on a client receiving dantrolene sodium
(Dantrium). Which laboratory test would identify an adverse effect associated with the
administration of this medication?
A. Creatinine

B. Liver function tests
C. Blood urea nitrogen
D. Hematological function tests
Answer: B. Liver function tests
Rationale:
Dose-related liver damage is the most serious adverse effect of dantrolene. To reduce the risk
of liver damage, liver function tests should be performed before treatment and periodically
throughout the treatment course. It is administered in the lowest effective dosage for the
shortest time necessary. *Eliminate options 1 and 3 because these tests both assess kidney
function.*
90. A nurse is reviewing the record of a client who has been prescribed baclofen (Lioresal).
Which of the following disorders, if noted in the client's history, would alert the nurse to
contact the health care provider?
A. Seizure disorders
B. Hyperthyroidism
C. Diabetes mellitus
D. Coronary artery disease
Answer: A. Seizure disorders
Rationale:
Clients with seizure disorders may have a lowered seizure threshold when baclofen is
administered. Concurrent therapy may require an increase in the anticonvulsive medication.
The disorders in options 2, 3, and 4 are not a concern when the client is taking baclofen
91. Cyclobenzaprine (Flexeril) is prescribed for a client to treat muscle spasms, and the nurse
is reviewing the client's record. Which of the following disorders, if noted in the client's
record, would indicate a need to contact the health care provider regarding the administration
of this medication?
A. Glaucoma
B. Emphysema
C. Hyperthyroidism
D. Diabetes mellitus
Answer: A. Glaucoma
Rationale:

Because this medication has anticholinergic effects, it should be used with caution in clients
with a history of urinary retention, angle-closure glaucoma, and increased intraocular
pressure. Cyclobenzaprine hydrochloride should be used only for short-term 2- to 3-week
therapy.
92. In monitoring a client's response to disease- modifying antirheumatic drugs (DMARDs),
which findings would the nurse interpret as acceptable responses? Select all that apply.
A. Symptom control during periods of emotional stress
B. Normal white blood cell counts, platelet, and neutrophil counts
C. Radiological findings that show nonprogression of joint degeneration
D. An increased range of motion in the affected joints 3 months into therapy
E. Inflammation and irritation at the injection site 3 days after injection is given
F. A low-grade temperature upon rising in the morning that remains throughout the day
Answer: A. Symptom control during periods of emotional stress
B. Normal white blood cell counts, platelet, and neutrophil counts
C. Radiological findings that show nonprogression of joint degeneration
D. An increased range of motion in the affected joints 3 months into therapy
Rationale:
Because emotional stress frequently exacerbates the symptoms of rheumatoid arthritis, the
absence of symptoms is a positive finding. DMARDs are given to slow progression of joint
degeneration. In addition, the improvement in the range of motion after 3 months of therapy
with normal blood work is a positive finding. Temperature elevation and inflammation and
irritation at the medication injection site could indicate signs of infection.
93. The client who is human immunodeficiency virus seropositive has been taking stavudine
(d4t, Zerit). The nurse monitors which of the following most closely while the client is taking
this medication?
A. Gait
B. Appetite
C. Level of consciousness
D. Hemoglobin and hematocrit blood levels
Answer: A. Gait
Rationale:

Stavudine (d4t, Zerit) is an antiretroviral used to manage human immunodeficiency virus
infection in clients who do not respond to or who cannot tolerate conventional therapy. The
medication can cause peripheral neuropathy, and the nurse should monitor the client's gait
closely and ask the client about paresthesia. Options 2, 3, and 4 are unrelated to the use of the
medication.
94. The client with acquired immunodeficiency syndrome has begun therapy with zidovudine
(Retrovir, Azidothymidine, AZT, ZDV). The nurse carefully monitors which of the following
laboratory results during treatment with this medication?
A. Blood culture
B. Blood glucose level
C. Blood urea nitrogen
D. Complete blood count
Answer: D. Complete blood count
Rationale:
A common side effect of therapy with zidovudine is leukopenia and anemia. The nurse
monitors the complete blood count results for these changes.Options 1, 2, and 3 are unrelated
to the use of this medication.
95. The nurse is reviewing the results of serum laboratory studies drawn on a client with
acquired immunodeficiency syndrome who is receiving didanosine (Videx). The nurse
interprets that the client may have the medication discontinued by the health care provider if
which of the following significantly elevated results is noted?
A. Serum protein
B. Blood glucose
C. Serum amylase
D. Serum creatinine
Answer: C. Serum amylase
Rationale:
Didanosine (Videx) can cause pancreatitis. A serum amylase level that is increased 1. 5 to 2
times normal may signify pancreatitis in the client with acquired immunodeficiency
syndrome and is potentially fatal. The medication may have to be discontinued. The
medication is also hepatotoxic and can result in liver failure.

96. The nurse is caring for a postrenal transplant client taking cyclosporine (Sandimmune,
Gengraf, Neoral). The nurse notes an increase in one of the client's vital signs, and the client
is complaining of a headache. What is the vital sign that is most likely increased?
A. Pulse
B. Respirations
C. Blood pressure
D. Pulse oximetry
Answer: C. Blood pressure
Rationale:
Hypertension can occur in a client taking cyclosporine (Sandimmune, Gengraf, Neoral), and
because this client is also complaining of a headache, the blood pressure is the vital sign to be
monitoring most closely. Other adverse effects include infection, nephrotoxicity, and
hirsutism. Options 1, 2, and 4 are unrelated to the use of this medication.
97. Amikacin (Amikin) is prescribed for a client with a bacterial infection. The client is
instructed to contact the health care provider (HCP) immediately if which of the following
occurs?
A. Nausea
B. Lethargy
C. Hearing loss
D. Muscle aches
Answer: C. Hearing loss
Rationale:
Amikacin (Amikin) is an aminoglycoside. Adverse effects of aminoglycosides include
ototoxicity (hearing problems), confusion, disorientation, gastrointestinal irritation,
palpitations, blood pressure changes, nephrotoxicity, and hypersensitivity. The nurse instructs
the client to report hearing loss to the HCP immediately. Lethargy and muscle aches are not
associated with the use of this medication. It is not necessary to contact the HCP immediately
if nausea occurs. If nausea persists or results in vomiting, the HCP should be notified. *(most
aminoglycoside medication names end in the letters -cin)*
98. The nurse is assigned to care for a client with cytomegalovirus retinitis and acquired
immunodeficiency syndrome who is receiving foscarnet. The nurse should check the latest
results of which of the following laboratory studies while the client is taking this medication?

A. CD4 cell count
B. Serum albumin
C. Serum creatinine
D. Lymphocyte count
Answer: C. Serum creatinine
Rationale:
Foscarnet is toxic to the kidneys. Serum creatinine is monitored before therapy, two to three
times per week during induction therapy, and at least weekly during maintenance therapy.
Foscarnet may also cause decreased levels of calcium, magnesium, phosphorus, and
potassium. Thus these levels are also measured with the same frequency.
99. The client with acquired immunodeficiency syndrome and Pneumocystis jiroveci
infection has been receiving pentamidine isethionate (Pentam 300). The client develops a
temperature of 101° F. The nurse does further monitoring of the client, knowing that this sign
would most likely indicate:
A. The dose of the medication is too low.
B. The client is experiencing toxic effects of the medication.
C. The client has developed inadequacy of thermoregulation.
D. The result of another infection caused by leukopenic effects of the medication.
Answer: D. The result of another infection caused by leukopenic effects of the medication.
Rationale:
Frequent side effects of this medication include leukopenia, thrombocytopenia, and anemia.
The client should be monitored routinely for signs and symptoms of infection. Options 1, 2,
and 3 are inaccurate interpretations.
100. Saquinavir (Invirase) is prescribed for the client who is human immunodeficiency virus
seropositive. The nurse reinforces medication instructions and tells the client to:
A. Avoid sun exposure.
B. Eat low-calorie foods.
C. Eat foods that are low in fat.
D. Take the medication on an empty stomach.
Answer: A. Avoid sun exposure.
Rationale:

Saquinavir (Invirase) is an antiretroviral (protease inhibitor) used with other antiretroviral
medications to manage human immunodeficiency virus infection. Saquinavir is administered
with meals and is best absorbed if the client consumes high-calorie, high-fat meals.
Saquinavir can cause photosensitivity, and the nurse should instruct the client to avoid sun
exposure.
101. Ketoconazole is prescribed for a client with a diagnosis of candidiasis. Select the
interventions that the nurse includes when administering this medication. Select all that apply.
A. Restrict fluid intake.
B. Instruct the client to avoid alcohol.
C. Monitor hepatic and liver function studies.
D. Administer the medication with an antacid.
E. Instruct the client to avoid exposure to the sun.
F. Administer the medication on an empty stomach.
Answer: B. Instruct the client to avoid alcohol.
C. Monitor hepatic and liver function studies.
E. Instruct the client to avoid exposure to the sun.
Rationale:
Ketoconazole is an antifungal medication. It is administered with food (not on an empty
stomach) and antacids are avoided for 2 hours after taking the medication to ensure
absorption. The medication is hepatotoxic and the nurse monitors liver function studies. The
client is instructed to avoid exposure to the sun because the medication increases
photosensitivity. The client is also instructed to avoid alcohol. There is no reason for the
client to restrict fluid intake. In fact, this could be harmful to the client.
102. A client with human immunodeficiency virus is taking nevirapine (Viramune). The nurse
should monitor for which adverse effects of the medication? Select all that apply.
A. Rash
B. Hepatotoxicity
C. Hyperglycemia
D. Peripheral neuropathy
E. Reduced bone mineral density
Answer: A. Rash
B. Hepatotoxicity

Rationale:
Nevirapine (Viramune) is a non-nucleoside reverse transcriptase inhibitors (NRTI) that is
used to treat HIV infection. It is used in combination with other antiretroviral medications to
treat HIV. Adverse effects include rash, Stevens- Johnson syndrome, hepatitis, and increased
transaminase levels. Hyperglycemia, peripheral neuropathy, and reduced bone density are not
adverse effects of this medication.
103. A nurse is caring for a hospitalized client who has been taking clozapine (Clozaril) for
the treatment of a schizophrenic disorder. Which laboratory study prescribed for the client
will the nurse specifically review to monitor for an adverse effect associated with the use of
this medication?
A. Platelet count
B. Cholesterol level
C. White blood cell count
D. Blood urea nitrogen level
Answer: C. White blood cell count
Rationale:
Hematological reactions can occur in the client taking clozapine and include agranulocytosis
and mild leukopenia. The white blood cell count should be checked before initiating
treatment and should be monitored closely during the use of this medication. The client
should also be monitored for signs indicating agranulocytosis, which may include sore throat,
malaise, and fever. Options 1, 2, and 4 are unrelated to this medication.
104. Disulfiram (Antabuse) is prescribed for a client who is seen in the psychiatric health care
clinic. The nurse is collecting data on the client and is providing instructions regarding the
use of this medication. Which is most important for the nurse to determine before
administration of this medication?
A. A history of hyperthyroidism
B. A history of diabetes insipidus
C. When the last full meal was consumed
D. When the last alcoholic drink was consumed
Answer: D. When the last alcoholic drink was consumed
Rationale:

Disulfiram is used as an adjunct treatment for selected clients with chronic alcoholism who
want to remain in a state of enforced sobriety. Clients must abstain from alcohol intake for at
least 12 hours before the initial dose of the medication is administered. The most important
data are to determine when the last alcoholic drink was consumed. The medication is used
with caution in clients with diabetes mellitus, hypothyroidism, epilepsy, cerebral damage,
nephritis, and hepatic disease. It is also contraindicated in severe heart disease, psychosis, or
hypersensitivity related to the medication.
105. A nurse is collecting data from a client and the client's spouse reports that the client is
taking donepezil hydrochloride (Aricept). Which disorder would the nurse suspect that this
client may have based on the use of this medication?
A. Dementia
B. Schizophrenia
C. Seizure disorder
D. Obsessive-compulsive disorder
Answer: A. Dementia
Rationale:
Donepezil hydrochloride is a cholinergic agent used in the treatment of mild to moderate
dementia of the Alzheimer type. It enhances cholinergic functions by increasing the
concentration of acetylcholine. It slows the progression of Alzheimer's disease. Options 2, 3,
and 4 are incorrect.
106. Fluoxetine (Prozac) is prescribed for the client. The nurse reinforces instructions to the
client regarding the administration of the medication. Which statement by the client indicates
an understanding about administration of the medication?
A. "I should take the medication with my evening meal."
B. "I should take the medication at noon with an antacid."
C. "I should take the medication in the morning when I first arise."
D. "I should take the medication right before bedtime with a snack."
Answer: C. "I should take the medication in the morning when I first arise."
Rationale:
Fluoxetine hydrochloride is administered in the early morning without consideration to
meals. *Eliminate options 1, 2, and 4 because they are comparable or alike and indicate
taking the medication with an antacid or food.*

107. A client receiving a tricyclic antidepressant arrives at the mental health clinic. Which
observation indicates that the client is correctly following the medication plan?
A. Reports not going to work for this past week
B. Complains of not being able to "do anything" anymore
C. Arrives at the clinic neat and appropriate in appearance
D. Reports sleeping 12 hours per night and 3 to 4 hours during the day
Answer: C. Arrives at the clinic neat and appropriate in appearance
Rationale:
Depressed individuals will sleep for long periods, are not able to go to work, and feel as if
they cannot "do anything." Once they have had some therapeutic effect from their
medication, they will report resolution of many of these complaints as well as demonstrate an
improvement in their appearance.
108. A nurse is performing a follow-up teaching session with a client discharged 1 month ago
who is taking fluoxetine (Prozac). What information would be important for the nurse to
gather regarding the adverse effects related to the medication?
A. Cardiovascular symptoms
B. Gastrointestinal dysfunctions
C. Problems with mouth dryness
D. Problems with excessive sweating
Answer: B. Gastrointestinal dysfunctions
Rationale:
The most common adverse effects related to fluoxetine include central nervous system (CNS)
and gastrointestinal (GI) system dysfunction. This medication affects the GI system by
causing nausea and vomiting, cramping, and diarrhea. Options 1, 3, and 4 are not adverse
effects of this medication.
109. A client taking buspirone (BuSpar) for 1 month returns to the clinic for a follow-up visit.
Which of the following would indicate medication effectiveness?
A. No rapid heartbeats or anxiety
B. No paranoid thought processes
C. No thought broadcasting or delusions
D. No reports of alcohol withdrawal symptoms

Answer: A. No rapid heartbeats or anxiety
Rationale:
Buspirone hydrochloride is not recommended for the treatment of drug or alcohol
withdrawal, paranoid thought disorders, or schizophrenia (thought broadcasting or delusions).
Buspirone hydrochloride is most often indicated for the treatment of anxiety and aggression.
110. A client taking lithium carbonate (Lithobid) reports vomiting, abdominal pain, diarrhea,
blurred vision, tinnitus, and tremors. The lithium level is checked as a part of the routine
follow-up and the level is 3. 0 mEq/L. The nurse knows that this level is:
A. Toxic
B. Normal
C. Slightly above normal
D. Excessively below normal
Answer: A. Toxic
Rationale:
The therapeutic serum level of lithium is 0.6 to 1.2 mEq/L. A level of 3 mEq/L indicates
toxicity.
111. A client arrives at the health care clinic and tells the nurse that he has been doubling his
daily dosage of bupropion hydrochloride (Wellbutrin) to help him get better faster. The nurse
understands that the client is now at risk for which of the following?
A. Insomnia
B. Weight gain
C. Seizure activity
D. Orthostatic hypotension .
Answer: C. Seizure activity
Rationale:
Bupropion does not cause significant orthostatic blood pressure changes. Seizure activity is
common in dosages greater than 450 mg daily. Bupropion frequently causes a drop in body
weight. Insomnia is a side effect, but seizure activity causes a greater client risk.
112. A hospitalized client is started on phenelzine sulfate (Nardil) for the treatment of
depression. The nurse instructs the client to avoid consuming which foods while taking this
medication? Select all that apply.

A. Figs
B. Yogurt
C. Crackers
D. Aged cheese
E Tossed salad
F. Oatmeal cookies
Answer: A. Figs
B. Yogurt
D. Aged cheese
Rationale:
Phenelzine sulfate (Nardil) is a monoamine oxidase inhibitor(MAOI). The client should avoid
taking in foods that are high in tyramine. Use of these foods could trigger a potentially fatal
hypertensive crisis. Some foods to avoid include yogurt, aged cheeses, smoked or processed
meats, red wines, and fruits such as avocados, raisins, and figs.
113. A nurse is reinforcing discharge instructions to a client receiving sulfisoxazole. Which of
the following would be included in the plan of care for instructions?
A. Maintain a high fluid intake.
B. Discontinue the medication when feeling better.
C. If the urine turns dark brown, call the health care provider immediately.
D. Decrease the dosage when symptoms are improving to prevent an allergic response.
Answer: A. Maintain a high fluid intake.
Rationale:
Each dose of sulfisoxazole should be administered with a full glass of water, and the client
should maintain a high fluid intake. The medication is more soluble in alkaline urine. The
client should not be instructed to taper or discontinue the dose. Some forms of sulfisoxazole
cause the urine to turn dark brown or red. This does not indicate the need to notify the health
care provider.
114. A postoperative client requests medication for flatulence (gas pains). Which medication
from the following PRN list should the nurse administer to this client?
A. Ondansetron (Zofran)
B. Simethicone (Mylicon)
C. Acetaminophen (Tylenol)

D. Magnesium hydroxide (milk of magnesia, MOM)
Answer: B. Simethicone (Mylicon)
Rationale:
Simethicone is an antiflatulent used in the relief of pain caused by excessive gas in the
gastrointestinal tract. Ondansetron is used to treat postoperative nausea and vomiting.
Acetaminophen is a nonopioid analgesic. Magnesium hydroxide is an antacid and laxative.
115. A client received 20 units of NPH insulin subcutaneously at 8:00 AM. The nurse should
check the client for a potential hypoglycemic reaction at what time?
A. 5:00 PM
B. 10:00 AM
C. 11:00 AM
D. 11:00 PM
Answer: A. 5:00 PM
Rationale:
NPH is intermediate-acting insulin. Its onset of action is 1 to 2½ hours, it peaks in 4 to 12
hours, and its duration of action is 24 hours. Hypoglycemic reactions most likely occur
during peak time.
116. A nurse administers a dose of scopolamine (Transderm-Scop) to a postoperative client.
The nurse tells the client to expect which of the following side effects of this medication?
A. Dry mouth
B. Diaphoresis
C. Excessive urination
D. Pupillary constriction
Answer: A. Dry mouth
Rationale:
Scopolamine is an anticholinergic medication for the prevention of nausea and vomiting that
causes the frequent side effects of dry mouth, urinary retention, decreased sweating, and
dilation of the pupils. The other options describe the opposite effects of cholinergic-blocking
agents and therefore are incorrect.

117. A nurse has given the client taking ethambutol (Myambutol) information about the
medication. The nurse determines that the client understands the instructions if the client
immediately reports:
A. Impaired sense of hearing
B. Distressing gastrointestinal side effects
C. Orange-red discoloration of body secretions
D. Difficulty discriminating the color red from green
Answer: D. Difficulty discriminating the color red from green
Rationale:
Ethambutol causes optic neuritis, which decreases visual acuity and the ability to discriminate
between the colors red and green. This poses a potential safety hazard when driving a motor
vehicle. The client is taught to report this symptom immediately. The client is also taught to
take the medication with food if gastrointestinal upset occurs. Impaired hearing results from
antitubercular therapy with streptomycin. Orange-red discoloration of secretions occurs with
rifampin (Rifadin).
118. A nurse is caring for an older client with a diagnosis of myasthenia gravis and has
reinforced self-care instructions. Which statement by the client indicates that further teaching
is necessary?
A. "I rest each afternoon after my walk."
B. "I cough and deep breathe many times during the day."
C. "If I get abdominal cramps and diarrhea, I should call my doctor."
D. "I can change the time of my medication on the mornings that I feel strong."
Answer: D. "I can change the time of my medication on the mornings that I feel strong."
Rationale:
The client with myasthenia gravis should be taught that timing of anticholinesterase
medication is critical. It is important to instruct the client to administer the medication on
time to maintain a chemical balance at the neuromuscular junction. If not given on time, the
client may become too weak to swallow. Options 1, 2, and 3 include the necessary
information that the client needs to understand to maintain health with this neurological
degenerative disease.
119. A client with diabetes mellitus who has been controlled with daily insulin has been
placed on atenolol (Tenormin) for the control of angina pectoris. Because of the effects of

atenolol, the nurse determines that which of the following is the most reliable indicator of
hypoglycemia?
A. Sweating
B. Tachycardia
C. Nervousness
D. Low blood glucose level
Answer: D. Low blood glucose level
Rationale:
β-Adrenergic blocking agents, such as atenolol, inhibit the appearance of signs and symptoms
of acute hypoglycemia, which would include nervousness, increased heart rate, and sweating.
Therefore, the client receiving this medication should adhere to the therapeutic regimen and
monitor blood glucose levels carefully. Option 4 is the most reliable indicator of
hypoglycemia.
120. A client is taking lansoprazole (Prevacid) for the chronic management of ZollingerEllison syndrome. The nurse advises the client to take which of the following products if
needed for a headache?
A. Naprosyn (Aleve)
B. Ibuprofen (Advil)
C. Acetaminophen (Tylenol)
D. Acetylsalicylic acid (aspirin) .
Answer: C. Acetaminophen (Tylenol)
Rationale:
Zollinger-Ellison syndrome is a hypersecretory condition of the stomach. The client should
avoid taking medications that are irritating to the stomach lining. Irritants would include
aspirin and nonsteroidal antiinflammatory drugs (ibuprofen). The client should be advised to
take acetaminophen for headache. *Remember that options that are comparable or alike are
not likely to be correct. With this in mind, eliminate options 1 and 2 first.*
121. A client who is taking hydrochlorothiazide (HydroDIURIL, HCTZ) has been started on
triamterene (Dyrenium) as well. The client asks the nurse why both medications are required.
The nurse formulates a response, based on the understanding that:
A. Both are weak potassium-losing diuretics.
B. The combination of these medications prevents renal toxicity.

C. Hydrochlorothiazide is an expensive medication, so using a combination of diuretics is
cost-effective.
D. Triamterene is a potassium-sparing diuretic, whereas hydrochlorothiazide is a potassiumlosing diuretic.
Answer: D. Triamterene is a potassium-sparing diuretic, whereas hydrochlorothiazide is a
potassium-losing diuretic.
Rationale:
Potassium-sparing diuretics include amiloride (Midamor), spironolactone (Aldactone), and
triamterene (Dyrenium). They are weak diuretics that are used in combination with
potassium-losing diuretics. This combination is useful when medication and dietary
supplement of potassium is not appropriate. The use of two different diuretics does not
prevent renal toxicity. Hydrochlorothiazide is an effective and inexpensive generic form of
the thiazide classification of diuretics. *It is especially helpful to remember that
hydrochlorothiazide is a potassium- losing diuretic and triamterene is a potassium-sparing
diuretic*
122. A client who has begun taking fosinopril (Monopril) is very distressed, telling the nurse
that he cannot taste food normally since beginning the medication 2 weeks ago. The nurse
provides the best support to the client by:
A. Telling the client not to take the medication with food
B. Suggesting that the client taper the dose until taste returns to normal
C. Informing the client that impaired taste is expected and generally disappears in 2 to 3
months
D. Requesting that the health care provider (HCP) change the prescription to another brand of
angiotensin-converting enzyme (ACE) inhibitor
Answer: C. Informing the client that impaired taste is expected and generally disappears in 2
to 3 months
Rationale:
ACE inhibitors, such as fosinopril, cause temporary impairment of taste (dysgeusia). The
nurse can tell the client that this effect usually disappears in 2 to 3 months, even with
continued therapy, and provide nutritional counseling if appropriate to avoid weight loss.
Options 1, 2, and 4 are inappropriate actions. Taking this medication with or without food
does not affect absorption and action. The dosage should never be tapered without HCP
approval and the medication should never be stopped abruptly.

123. A nurse is planning to administer amlodipine (Norvasc) to a client. The nurse plans to
check which of the following before giving the medication?
A. Respiratory rate
B. Blood pressure and heart rate
C. Heart rate and respiratory rate
D. Level of consciousness and blood pressure
Answer: B. Blood pressure and heart rate
Rationale:
Amlodipine is a calcium channel blocker. This medication decreases the rate and force of
cardiac contraction. Before administering a calcium channel blocking agent, the nurse should
check the blood pressure and heart rate, which could both decrease in response to the action
of this medication. This action will help to prevent or identify early problems related to
decreased cardiac contractility, heart rate, and conduction. *amlodipine is a calcium channel
blocker, and this group of medications decreases the rate and force of cardiac contraction.
This in turn lowers the pulse rate and blood pressure.*
124. A client with chronic renal failure is receiving ferrous sulfate (Feosol). The nurse
monitors the client for which common side effect associated with this medication?
A. Diarrhea
B. Weakness
C. Headache
D. Constipation
Answer: D. Constipation
Rationale:
Feosol is an iron supplement used to treat anemia. Constipation is a frequent and
uncomfortable side effect associated with the administration of oral iron supplements. Stool
softeners are often prescribed to prevent constipation. *Focus on the name of the medication.
Recalling that oral iron can cause constipation will easily direct you to the correct option.*
125. A nurse is preparing to administer digoxin (Lanoxin), 0.125 mg orally, to a client with
heart failure. Which vital sign is most important for the nurse to check before administering
the medication?
A. Heart rate

B. Temperature
C. Respirations
D. Blood pressure
Answer: A. Heart rate
Rationale:
Digoxin is a cardiac glycoside that is used to treat heart failure and acts by increasing the
force of myocardial contraction. Because bradycardia may be a clinical sign of toxicity, the
nurse counts the apical heart rate for 1 full minute before administering the medication. If the
pulse rate is less than 60 beats/minute in an adult client, the nurse would withhold the
medication and report the pulse rate to the registered nurse, who would then contact the
health care provider.
126. A nurse is caring for a client who has been prescribed furosemide (Lasix) and is
monitoring for adverse effects associated with this medication. Which of the following should
the nurse recognize as a potential adverse effect Select all that apply.
A. Nausea
B. Tinnitus
C. Hypotension
D. Hypokalemia
E. Photosensitivity
F. Increased urinary frequency
Answer: B. Tinnitus
C. Hypotension
D. Hypokalemia
Rationale:
Furosemide is a loop diuretic; therefore, an expected effect is increased urinary frequency.
Nausea is a frequent side effect, not an adverse effect. Photosensitivity is an occasional side
effect. Adverse effects include tinnitus (ototoxicity), hypotension, and hypokalemia and occur
as a result of sudden volume depletion.
127. The nurse provides medication instructions to an older hypertensive client who is taking
20 mg of lisinopril (Prinivil, Zestril) orally daily. The nurse evaluates the need for further
teaching when the client states which of the following?
A. "I can skip a dose once a week."

B. "I need to change my position slowly."
C. "I take the pill after breakfast each day."
D. "If I get a bad headache, I should call my doctor immediately."
Answer: A. "I can skip a dose once a week."
Rationale:
Lisinopril is an antihypertensive angiotensin-converting enzyme (ACE) inhibitor. The usual
dosage range is 20 to 40 mg per day. Adverse effects include headache, dizziness, fatigue,
orthostatic hypotension, tachycardia, and angioedema. Specific client teaching points include
taking one pill a day, not stopping the medication without consulting the health care provider
(HCP), and monitoring for side effects and adverse reactions. The client should notify the
HCP if side effects occur.
128. A nurse is providing instructions to an adolescent who has a history of seizures and is
taking an anticonvulsant medication. Which of the following statements indicates that the
client understands the instructions?
A. "I will never be able to drive a car."
B. "My anticonvulsant medication will clear up my skin."
C. "I can't drink alcohol while I am taking my medication."
D. "If I forget my morning medication, I can take two pills at bedtime."
Answer: C. "I can't drink alcohol while I am taking my medication."
Rationale:
Alcohol will lower the seizure threshold and should be avoided. Adolescents can obtain a
driver's license in most states when they have been seizure free for 1 year. Anticonvulsants
cause acne and oily skin; therefore a dermatologist may need to be consulted. If an
anticonvulsant medication is missed, the health care provider should be notified.
129. Megestrol acetate (Megace), an antineoplastic medication, is prescribed for the client
with metastatic endometrial carcinoma. The nurse reviews the client's history and contacts the
registered nurse if which diagnosis is documented in the client's history?
A. Gout
B. Asthma
C. Thrombophlebitis
D. Myocardial infarction
Answer: C. Thrombophlebitis

Rationale:
Megestrol acetate (Megace) suppresses the release of luteinizing hormone from the anterior
pituitary by inhibiting pituitary function and regressing tumor size. Megestrol is used with
caution if the client has a history of thrombophlebitis. *megestrol acetate is a hormonal
antagonist enzyme and that a side effect is thrombotic disorders*
130. The nurse is analyzing the laboratory results of a client with leukemia who has received
a regimen of chemotherapy. Which laboratory value would the nurse specifically note as a
result of the massive cell destruction that occurred from the chemotherapy?
A. Anemia
B. Decreased platelets
C. Increased uric acid level
D. Decreased leukocyte count
Answer: C. Increased uric acid level
Rationale:
Hyperuricemia is especially common following treatment for leukemias and lymphomas
because chemotherapy results in a massive cell kill. Although options 1, 2, and 4 also may be
noted, an increased uric acid level is related specifically to cell destruction.
131. The nurse is reinforcing medication instructions to a client with breast cancer who is
receiving cyclophosphamide (Neosar). The nurse tells the client to:
A. Take the medication with food.
B. Increase fluid intake to 2000 to 3000 mL daily.
C. Decrease sodium intake while taking the medication.
D. Increase potassium intake while taking the medication.
Answer: B. Increase fluid intake to 2000 to 3000 mL daily.
Rationale:
Hemorrhagic cystitis is a toxic effect that can occur with the use of cyclophosphamide. The
client needs to be instructed to drink copious amounts of fluid during the administration of
this medication. Clients also should monitor urine output for hematuria. The medication
should be taken on an empty stomach, unless gastrointestinal (GI) upset occurs.
Hyperkalemia can result from the use of the medication; therefore the client would not be
told to increase potassium intake. The client would not be instructed to alter sodium intake.

132. The client with non-Hodgkin's lymphoma is receiving daunorubicin (DaunoXome).
Which of the following would indicate to the nurse that the client is experiencing a toxic
effect related to the medication?
A. Fever
B. Diarrhea
C. Complaints of nausea and vomiting
D. Crackles on auscultation of the lungs
Answer: D. Crackles on auscultation of the lungs
Rationale:
Cardiotoxicity noted by abnormal electrocardiographic findings or cardiomyopathy
manifested as congestive heart failure is a toxic effect of daunorubicin. Bone marrow
depression is also a toxic effect. Nausea and vomiting are frequent side effects associated
with the medication that begins a few hours after administration and lasts 24 to 48 hours.
Fever is a frequent side effect, and diarrhea can occur occasionally. The other options,
however, are not toxic effects. *keep in mind that the question is asking about a toxic effect
and think: ABCs— airway, breathing, and circulation*
133. A nurse is monitoring a client receiving desmopressin acetate (DDAVP) for adverse
effects to the medication. Which of the following indicates the presence of an adverse effect?
A. Insomnia
B. Drowsiness
C. Weight loss
D. Increased urination eliminate weight loss and increased urination.*
Answer: B. Drowsiness
Rationale:
Water intoxication (overhydration) or hyponatremia is an adverse effect to desmopressin.
Early signs include drowsiness, listlessness, and headache. Decreased urination, rapid weight
gain, confusion, seizures, and coma also may occur in overhydration. *Recall that this
medication is used to treat diabetes insipidus to eliminate weight loss and increased
urination.*
134. A nurse reinforces instructions to a client who is taking levothyroxine (Synthroid). The
nurse tells the client to take the medication:
A. With food

B. At lunchtime
C. On an empty stomach
D. At bedtime with a snack
Answer: C. On an empty stomach
Rationale:
Oral doses of levothyroxine (Synthroid) should be taken on an empty stomach to enhance
absorption. Dosing should be done in the morning before breakfast. Note that options 1, 2,
and 4 are comparable or alike in that these options address administering the medication with
food.
135. A nurse reinforces medication instructions to a client who is taking levothyroxine
(Synthroid). The nurse instructs the client to notify the health care provider (HCP) if which of
the following occurs?
A. Fatigue
B. Tremors
C. Cold intolerance
D. Excessively dry skin
Answer: B. Tremors
Rationale:
Excessive doses of levothyroxine (Synthroid) can produce signs and symptoms of
hyperthyroidism. These include tachycardia, chest pain, tremors, nervousness, insomnia,
hyperthermia, heat intolerance, and sweating. The client should be instructed to notify the
HCP if these occur. Options 1, 3, and 4 are signs of hypothyroidism.
136. A nurse performs an admission assessment on a client who visits a health care clinic for
the first time. The client tells the nurse that propylthiouracil (PTU) is taken daily. The nurse
continues to collect data from the client, suspecting that the client has a history of:
A. Myxedema
B. Graves' disease
C. Addison's disease
D. Cushing's syndrome
Answer: B. Graves' disease
Rationale:

PTU inhibits thyroid hormone synthesis and is used to treat hyperthyroidism, or Graves'
disease. Myxedema indicates hypothyroidism. Cushing's syndrome and Addison's disease are
disorders related to adrenal function.
137. A nurse is reinforcing instructions for a client regarding intranasal desmopressin acetate
(DDAVP). The nurse tells the client that which of the following is a side effect of the
medication?
A. Headache
B. Vulval pain
C. Runny nose
D. Flushed skin
Answer: C. Runny nose
Rationale:
Desmopressin administered by the intranasal route can cause a runny or stuffy nose.
Headache, vulval pain, and flushed skin are side effects if the medication is administered by
the intravenous (IV) route.
138. A daily dose of prednisone is prescribed for a client. A nurse reinforces instructions to
the client regarding administration of the medication and instructs the client that the best time
to take this medication is:
A. At noon
B. At bedtime
C. Early morning
D. Anytime, at the same time, each day
Answer: C. Early morning
Rationale:
Corticosteroids (glucocorticoids) should be administered before 9:00 AM. Administration at
this time helps minimize adrenal insufficiency and mimics the burst of glucocorticoids
released naturally by the adrenal glands each morning. *Note the suffix "-sone," and recall
that medication names that end with these letters are corticosteroids.* 3. An increased amount
of daily Humulin NPH

139. Prednisone is prescribed for a client with diabetes mellitus who is taking Humulin
neutral protamine Hagedorn (NPH) insulin daily. Which of the following prescription
changes does the nurse anticipate during therapy with the prednisone?
A. An additional dose of prednisone daily
B. A decreased amount of daily Humulin NPH insulin
C. An increased amount of daily Humulin NPH insulin
D. The addition of an oral hypoglycemic medication dail
Answer: C. An increased amount of daily Humulin NPH insulin
Rationale:
Glucocorticoids can elevate blood glucose levels. Clients with diabetes mellitus may need
their dosages of insulin or oral hypoglycemic medications increased during glucocorticoid
therapy. Therefore the other options are incorrect.
140. The client has a new prescription for metoclopramide (Reglan). On review of the chart,
the nurse identifies that this medication can be safely administered with which condition?
A. Intestinal obstruction
B. Peptic ulcer with melena
C. Diverticulitis with perforation
D. Vomiting following cancer chemotherapy
Answer: D. Vomiting following cancer chemotherapy
Rationale:
Metoclopramide is a gastrointestinal (GI) stimulant and antiemetic. Because it is a GI
stimulant, it is contraindicated with GI obstruction, hemorrhage, or perforation. It is used in
the treatment of emesis after surgery, chemotherapy, and radiation.
141. The nurse has reinforced instructions to a client who has been prescribed cholestyramine
(Questran). Which statement by the client indicates a need for further instructions?
A. "I will continue taking vitamin supplements."
B. "This medication will help lower my cholesterol."
C. "This medication should only be taken with water."
D. "A high-fiber diet is important while taking this medication."
Answer: C. "This medication should only be taken with water."
Rationale:

Cholestyramine (Questran) is a bile acid sequestrant used to lower the cholesterol level, and
client compliance is a problem because of its taste and palatability. The use of flavored
products or fruit juices can improve the taste. Some side effects of bile acid sequestrants
include constipation and decreased vitamin absorption. *Note the closed-ended word "only"
in option 3*
142. A health care provider has written a prescription for ranitidine (Zantac), once daily. The
nurse should schedule the medication for which of the following times?
A. At bedtime
B. After lunch
C. With supper
D. Before breakfast
Answer: A. At bedtime
Rationale:
A single daily dose of ranitidine is usually scheduled to be given at bedtime. This allows for a
prolonged effect, and the greatest protection of the gastric mucosa. *recall that ranitidine
suppresses secretions of gastric acids*
143. A client has just taken a dose of trimethobenzamide (Tigan). The nurse plans to monitor
this client for relief of:
A. Heartburn
B. Constipation
C. Abdominal pain
D. Nausea and vomiting
Answer: D. Nausea and vomiting
Rationale:
Trimethobenzamide is an antiemetic agent used in the treatment of nausea and vomiting. The
other options are incorrect.
144. A client is taking docusate sodium (Colace). The nurse monitors which of the following
to determine whether the client is having a therapeutic effect from this medication?
A. Abdominal pain
B. Reduction in steatorrhea
C. Hematest-negative stools

D. Regular bowel movements
Answer: D. Regular bowel movements
Rationale:
Docusate sodium is a stool softener that promotes the absorption of water into the stool,
producing a softer consistency of stool. The intended effect is relief or prevention of
constipation. The medication does not relieve abdominal pain, stop gastrointestinal (GI)
bleeding, or decrease the amount of fat in the stools.
145. A nurse has a prescription to give a client albuterol (Proventil HFA) (two puffs) and
beclomethasone dipropionate (Qvar) (nasal inhalation, two puffs), by metered- dose inhaler.
The nurse administers the medication by giving the:
A. Albuterol first and then the beclomethasone dipropionate
B. Beclomethasone dipropionate first and then the albuterol
C. Alternating a single puff of each, beginning with the albuterol
D. Alternating a single puff of each, beginning with the beclomethasone dipropionate 46.
Answer: A. Albuterol first and then the beclomethasone dipropionate
Rationale:
Albuterol is a bronchodilator. Beclomethasone dipropionate is a glucocorticoid.
Bronchodilators are always administered before glucocorticoids when both are to be given on
the same time schedule. This allows for widening of the air passages by the bronchodilator,
which then makes the glucocorticoid more effective.
146. A client has begun therapy with theophylline (Theo-24). The nurse tells the client to
limit the intake of which of the following while taking this medication?
A. Oranges and pineapple
B. Coffee, cola, and chocolate
C. Oysters, lobster, and shrimp
D. Cottage cheese, cream cheese, and dairy creamers
Answer: B. Coffee, cola, and chocolate
Rationale:
Theophylline is a xanthine bronchodilator. The nurse teaches the client to limit the intake of
xanthine-containing foods while taking this medication. These include coffee, cola, and
chocolate.

147. A client with a prescription to take theophylline (Theo-24) daily has been given
medication instructions by the nurse. The nurse determines that the client needs further
information about the medication if the client states that he or she will:
A. Drink at least 2 L of fluid per day.
B. Take the daily dose at bedtime.
C. Avoid changing brands of the medication without health care provider (HCP) approval.
D. Avoid over-the-counter (OTC) cough and cold medications unless approved by the HCP.
Answer: B. Take the daily dose at bedtime.
Rationale:
The client taking a single daily dose of theophylline, a xanthine bronchodilator, should take
the medication early in the morning. This enables the client to have maximal benefit from the
medication during daytime activities. In addition, this medication causes insomnia. The client
should take in at least 2 L of fluid per day to decrease viscosity of secretions. The client
should check with the physician before changing brands of the medication. The client also
checks with the HCP before taking OTC cough, cold, or other respiratory preparations
because they could cause interactive effects, increasing the side effects of theophylline and
causing dysrhythmias.
148. A client is taking cetirizine hydrochloride (Zyrtec). The nurse checks for which of the
following side effects of this medication?
A. Diarrhea
B. Excitability
C. Drowsiness
D. Excess salivation
Answer: C. Drowsiness
Rationale:
A frequent side effect of cetirizine hydrochloride (Zyrtec), an antihistamine, is drowsiness or
sedation. Others include blurred vision, hypertension (and sometimes hypotension), dry
mouth, constipation, urinary retention, and sweating.
149. A client taking fexofenadine (Allegra) is scheduled for allergy skin testing and tells the
nurse in the health care provider's office that a dose was taken this morning. The nurse
determines that:
A. The client should reschedule the appointment.

B. A lower dose of allergen will need to be injected.
C. A higher dose of allergen will need to be injected.
D. The client should have the skin test read a day later than usual.
Answer: A. The client should reschedule the appointment.
Rationale:
Fexofenadine is an antihistamine, which provides relief of symptoms caused by allergy.
Antihistamines should be discontinued for at least 3 days (72 hours) before allergy skin
testing to avoid false-negative readings. This client should have the appointment rescheduled
for 3 days after discontinuing the medication.
150. A client complaining of not feeling well is seen in a clinic. The client is taking several
medications for the control of heart disease and hypertension. These medications include a βblocker, digoxin (Lanoxin), and a diuretic. A tentative diagnosis of digoxin toxicity is made.
Which of the following assessment data would support this diagnosis?
A. Dyspnea, edema, and palpitations
B. Chest pain, hypotension, and paresthesia
C. Double vision, loss of appetite, and nausea
D. Constipation, dry mouth, and sleep disorder
Answer: C. Double vision, loss of appetite, and nausea
Rationale:
Double vision, loss of appetite, and nausea are signs of digoxin toxicity. Additional signs of
digoxin toxicity include bradycardia, difficulty reading, visual alterations such as green and
yellow vision or seeing spots or halos, confusion, vomiting, diarrhea, decreased libido, and
impotence. *gastrointestinal (GI) and visual disturbances occur with digoxin toxicity*
151. A client is being treated for acute congestive heart failure with intravenously
administered bumetanide. The vital signs are as follows: blood pressure, 100/60 mm Hg;
pulse, 96 beats/min; and respirations, 24 breaths/min. After the initial dose, which of the
following is the priority assessment?
A. Monitoring weight loss
B. Monitoring temperature
C. Monitoring blood pressure
D. Monitoring potassium level
Answer: C. Monitoring blood pressure

Rationale:
Bumetanide is a loop diuretic. Hypotension is a common side effect associated with the use
of this medication. The other options also require assessment but are not the priority. *priority
ABCs—airway, breathing, and circulation*
152. Intravenous heparin therapy is prescribed for a client. While implementing this
prescription, a nurse ensures that which of the following medications is available on the
nursing unit?
A. Protamine sulfate
B. Potassium chloride
C. Phytonadione (vitamin K )
D. Aminocaproic acid (Amicar)
Answer: A. Protamine sulfate
Rationale:
The antidote to heparin is protamine sulfate; it should be readily available for use if excessive
bleeding or hemorrhage occurs. Potassium chloride is administered for a potassium deficit.
Vitamin K is an antidote for warfarin sodium. Aminocaproic acid is the antidote for
thrombolytic therapy.
153. A client is diagnosed with pulmonary embolism and is to be treated with streptokinase
(Streptase). A nurse would report which priority data collection finding to the registered nurse
before initiating this therapy?
A. Adventitious breath sounds
B. Temperature of 99.4° F orally
C. Blood pressure of 198/110 mm Hg
D. Respiratory rate of 28 breaths/min 54.
Answer: C. Blood pressure of 198/110 mm Hg
Rationale:
Thrombolytic therapy is contraindicated in a number of preexisting conditions in which there
is a risk of uncontrolled bleeding, similar to the case in anticoagulant therapy. Thrombolytic
therapy also is contraindicated in severe uncontrolled hypertension because of the risk of
cerebral hemorrhage. Therefore the nurse would report the results of the blood pressure to the
registered nurse before initiating therapy. The findings in options 1, 2, and 4 may be present
in the client with pulmonary embolism.

154. ) A nurse is reinforcing dietary instructions to a client who has been prescribed
cyclosporine (Sandimmune). Which food item would the nurse instruct the client to avoid?
A. Red meats
B. Orange juice
C. Grapefruit juice
D. Green, leafy vegetables 5.
Answer: C. Grapefruit juice
Rationale:
A compound present in grapefruit juice inhibits metabolism of cyclosporine. As a result, the
consumption of grapefruit juice can raise cyclosporine levels by 50% to 100%, thereby
greatly increasing the risk of toxicity. Grapefruit juice needs to be avoided. Red meats,
orange juice, and green leafy vegetables are acceptable to consume.
155. Mycophenolate mofetil (CellCept) is prescribed for a client as prophylaxis for organ
rejection following an allogeneic renal transplant. Which of the following instructions does
the nurse reinforce regarding administration of this medication?
A. Administer following meals.
B. Take the medication with a magnesium-type antacid.
C. Open the capsule and mix with food for administration.
D. Contact the health care provider (HCP) if a sore throat occurs.
Answer: D. Contact the health care provider (HCP) if a sore throat occurs.
Rationale:
Mycophenolate mofetil should be administered on an empty stomach. The capsules should
not be opened or crushed. The client should contact the HCP if unusual bleeding or bruising,
sore throat, mouth sores, abdominal pain, or fever occurs because these are adverse effects of
the medication. Antacids containing magnesium and aluminum may decrease the absorption
of the medication and therefore should not be taken with the medication. The medication may
be given in combination with corticosteroids and cyclosporine. *neutropenia can occur with
this medication*
156. A nurse is reviewing the laboratory results for a client receiving tacrolimus (Prograf).
Which laboratory result would indicate to the nurse that the client is experiencing an adverse
effect of the medication?

A. Blood glucose of 200 mg/dL
B. Potassium level of 3. 8 mEq/L
C. Platelet count of 300,000 cells/mm3
D. White blood cell count of 6000 cells/mm3
Answer: A. Blood glucose of 200 mg/dL
Rationale:
A blood glucose level of 200 mg/dL is elevated above the normal range of 70 to 110 mg/dL
and suggests an adverse effect. Other adverse effects include neurotoxicity evidenced by
headache, tremor, insomnia; gastrointestinal (GI) effects such as diarrhea, nausea, and
vomiting; hypertension; and hyperkalemia.
157. A client receiving nitrofurantoin (Macrodantin) calls the health care provider's office
complaining of side effects related to the medication. Which side effect indicates the need to
stop treatment with this medication?
A. Nausea
B. Diarrhea
C. Anorexia
D. Cough and chest pain
Answer: D. Cough and chest pain
Rationale:
Gastrointestinal (GI) effects are the most frequent adverse reactions to this medication and
can be minimized by administering the medication with milk or meals. Pulmonary reactions,
manifested as dyspnea, chest pain, chills, fever, cough, and the presence of alveolar infiltrates
on the x-ray, would indicate the need to stop the treatment. These symptoms resolve in 2 to 4
days following discontinuation of this medication. *Eliminate options 1, 2, and 3 because
they are similar GI-related side effects. Also, use the ABCs— airway, breathing, and
circulation*
158. A client with chronic renal failure is receiving epoetin alfa (Epogen, Procrit). Which
laboratory result would indicate a therapeutic effect of the medication?
A. Hematocrit of 32%
B. Platelet count of 400,000 cells/mm3
C. White blood cell count of 6000 cells/mm3
D. Blood urea nitrogen (BUN) level of 15 mg/dL

Answer: A. Hematocrit of 32%
Rationale:
Epoetin alfa is used to reverse anemia associated with chronic renal failure. A therapeutic
effect is seen when the hematocrit is between 30% and 33%. The laboratory tests noted in the
other options are unrelated to the use of this medication.
159. A nurse is caring for a client receiving morphine sulfate subcutaneously for pain.
Because morphine sulfate has been prescribed for this client, which nursing action would be
included in the plan of care?
A. Encourage fluid intake.
B. Monitor the client's temperature.
C. Maintain the client in a supine position.
D. Encourage the client to cough and deep breathe.
Answer: D. Encourage the client to cough and deep breathe.
Rationale:
Morphine sulfate suppresses the cough reflex. Clients need to be encouraged to cough and
deep breathe to prevent pneumonia. *ABCs—airway, breathing, and circulation*
160. Meperidine hydrochloride (Demerol) is prescribed for the client with pain. Which of the
following would the nurse monitor for as a side effect of this medication?
A. Diarrhea
B. Bradycardia
C. Hypertension
D. Urinary retention
Answer: D. Urinary retention
Rationale:
Meperidine hydrochloride (Demerol) is an opioid analgesic. Side effects of this medication
include respiratory depression, orthostatic hypotension, tachycardia, drowsiness and mental
clouding, constipation, and urinary retention.
161. A nurse is caring for a client with severe back pain, and codeine sulfate has been
prescribed for the client. Which of the following would the nurse include in the plan of care
while the client is taking this medication?
A. Restrict fluid intake.

B. Monitor bowel activity.
C. Monitor for hypertension.
D. Monitor peripheral pulses.
Answer: B. Monitor bowel activity.
Rationale:
While the client is taking codeine sulfate, an opioid analgesic, the nurse would monitor vital
signs and monitor for hypotension. The nurse should also increase fluid intake, palpate the
bladder for urinary retention, auscultate bowel sounds, and monitor the pattern of daily bowel
activity and stool consistency (codeine can cause constipation). The nurse should monitor
respiratory status and initiate breathing and coughing exercises. In addition, the nurse
monitors the effectiveness of the pain medication.
162. Carbamazepine (Tegretol) is prescribed for a client with a diagnosis of psychomotor
seizures. The nurse reviews the client's health history, knowing that this medication is
contraindicated if which of the following disorders is present?
A. Headaches
B. Liver disease
C. Hypothyroidism
D. Diabetes mellitus
Answer: B. Liver disease
Rationale:
Carbamazepine (Tegretol) is contraindicated in liver disease, and liver function tests are
routinely prescribed for baseline purposes and are monitored during therapy. It is also
contraindicated if the client has a history of blood dyscrasias. It is not contraindicated in the
conditions noted in the incorrect options.
163. A client with trigeminal neuralgia tells the nurse that acetaminophen (Tylenol) is taken
on a frequent daily basis for relief of generalized discomfort. The nurse reviews the client's
laboratory results and determines that which of the following indicates toxicity associated
with the medication?
A. Sodium of 140 mEq/L
B. Prothrombin time of 12 seconds
C. Platelet count of 400,000 cells/mm3
D. A direct bilirubin level of 2 mg/dL

Answer: D. A direct bilirubin level of 2 mg/dL
Rationale:
In adults, overdose of acetaminophen (Tylenol) causes liver damage. Option 4 is an indicator
of liver function and is the only option that indicates an abnormal laboratory value. The
normal direct bilirubin is 0 to 0.4 mg/dL. The normal platelet count is 150,000 to 400,000
cells/mm3. The normal prothrombin time is 10 to 13 seconds. The normal sodium level is 135
to 145 mEq/L.
164. A client receives a prescription for methocarbamol (Robaxin), and the nurse reinforces
instructions to the client regarding the medication. Which client statement would indicate a
need for further instructions?
A. "My urine may turn brown or green."
B. "This medication is prescribed to help relieve my muscle spasms."
C. "If my vision becomes blurred, I don't need to be concerned about it."
D. "I need to call my doctor if I experience nasal congestion from this medication."
Answer: C. "If my vision becomes blurred, I don't need to be concerned about it."
Rationale:
The client needs to be told that the urine may turn brown, black, or green. Other adverse
effects include blurred vision, nasal congestion, urticaria, and rash. The client needs to be
instructed that, if these adverse effects occur, the health care provider needs to be notified.
The medication is used to relieve muscle spasms.
165. The client has been on treatment for rheumatoid arthritis for 3 weeks. During the
administration of etanercept (Enbrel), it is most important for the nurse to assess:
A. The injection site for itching and edema
B. The white blood cell counts and platelet counts
C. Whether the client is experiencing fatigue and joint pain
D. A metallic taste in the mouth and a loss of appetite
Answer: B. The white blood cell counts and platelet counts
Rationale:
Infection and pancytopenia are adverse effects of etanercept (Enbrel). Laboratory studies are
performed before and during treatment. The appearance of abnormal white blood cell counts
and abnormal platelet counts can alert the nurse to a potential life- threatening infection.
Injection site itching is a common occurrence following administration of the medication. In

early treatment, residual fatigue and joint pain may still be apparent. A metallic taste and loss
of appetite are not common signs of side effects of this medication.
166. Alendronate (Fosamax) is prescribed for a client with osteoporosis. The client taking this
medication is instructed to:
A. Take the medication at bedtime.
B. Take the medication in the morning with breakfast.
C. Lie down for 30 minutes after taking the medication.
D. Take the medication with a full glass of water after rising in the morning.
Answer: D. Take the medication with a full glass of water after rising in the morning.
Rationale:
Precautions need to be taken with administration of alendronate to prevent gastrointestinal
side effects (especially esophageal irritation) and to increase absorption of the medication.
The medication needs to be taken with a full glass of water after rising in the morning. The
client should not eat or drink anything for 30 minutes following administration and should
not lie down after taking the medication.
167. A nurse prepares to reinforce instructions to a client who is taking allopurinol
(Zyloprim). The nurse plans to include which of the following in the instructions?
A. Instruct the client to drink 3000 mL of fluid per day.
B. Instruct the client to take the medication on an empty stomach.
C. Inform the client that the effect of the medication will occur immediately.
D. Instruct the client that, if swelling of the lips occurs, this is a normal expected response.
Answer: A. I instructs e client to drink 3000 mL of fluid per day.
Rationale:
Allopurinol (Zyloprim) is an antigout medication used to decrease uric acid levels. Clients
taking allopurinol are encouraged to drink 3000 mL of fluid a day. A full therapeutic effect
may take 1 week or longer. Allopurinol is to be given with or immediately following meals or
milk to prevent gastrointestinal irritation. If the client develops a rash, irritation of the eyes,
or swelling of the lips or mouth, he or she should contact the health care provider because this
may indicate hypersensitivity.

168. Colcrys (colchicine) is prescribed for a client with a diagnosis of gout. The nurse
reviews the client's medical history in the health record, knowing that the medication would
be contraindicated in which disorder?
A. Myxedema
B. Renal failure
C. Hypothyroidism
D. Diabetes mellitus
Answer: B. Renal failure
Rationale:
Colchicine is contraindicated in clients with severe gastrointestinal, renal, hepatic or cardiac
disorders, or with blood dyscrasias. Clients with impaired renal function may exhibit
myopathy and neuropathy manifested as generalized weakness. This medication should be
used with caution in clients with impaired hepatic function, older clients, and debilitated
clients. *Note that options 1, 3, and 4 are all endocrine-related disorders:
Myxedema=Hypothyroidism*
169. Insulin glargine (Lantus) is prescribed for a client with diabetes mellitus. The nurse tells
the client that it is best to take the insulin:
A. 1 hour after each meal
B. Once daily, at the same time each day
C. 15 minutes before breakfast, lunch, and dinner
D. Before each meal, on the basis of the blood glucose level
Answer: B. Once daily, at the same time each day
Rationale:
Insulin glargine is a long-acting recombinant DNA human insulin used to treat type 1 and
type 2 diabetes mellitus. It has a 24-hour duration of action and is administered once a day, at
the same time each day.
170. Atenolol hydrochloride (Tenormin) is prescribed for a hospitalized client. The nurse
should perform which of the following as a priority action before administering the
medication?
A. Listen to the client's lung sounds.
B. Check the client's blood pressure.
C. Check the recent electrolyte levels.

D. Assess the client for muscle weakness.
Answer: B. Check the client's blood pressure.
Rationale:
Atenolol hydrochloride is a beta-blocker used to treat hypertension. Therefore the priority
nursing action before administration of the medication is to check the client's blood pressure.
The nurse also checks the client's apical heart rate. If the systolic blood pressure is below 90
mm Hg or the apical pulse is 60 beats per minute or lower, the medication is withheld and the
registered nurse and/or health care provider is notified. The nurse would check baseline renal
and liver function tests. The medication may cause weakness, and the nurse would assist the
client with activities if weakness occurs. *Beta-blockers have "-lol" at the end of the
medication name*
171. A nurse is preparing to administer furosemide (Lasix) to a client with a diagnosis of
heart failure. The most important laboratory test result for the nurse to check before
administering this medication is:
A. Potassium level
B. Creatinine level
C. Cholesterol level
D. Blood urea nitrogen
Answer: A. Potassium level
Rationale:
Furosemide is a loop diuretic. The medication causes a decrease in the client's electrolytes,
especially potassium, sodium, and chloride. Administering furosemide to a client with low
electrolyte levels could precipitate ventricular dysrhythmias. Options 2 and 4 reflect renal
function. The cholesterol level is unrelated to the administration of this medication.
172. A nurse provides dietary instructions to a client who will be taking warfarin sodium
(Coumadin). The nurse tells the client to avoid which food
A. Grapes
B. Spinach
C. Watermelon
D. Cottage cheese
Answer: B. Spinach
Rationale:

Warfarin sodium is an anticoagulant. Anticoagulant medications act by antagonizing the
action of vitamin K, which is needed for clotting. When a client is taking an anticoagulant,
foods high in vitamin K often are omitted from the diet. Vitamin K-rich foods include green,
leafy vegetables, fish, liver, coffee, and tea.
173. A nurse reviews the medication history of a client admitted to the hospital and notes that
the client is taking leflunomide (Arava). During data collection, the nurse asks which
question to determine medication effectiveness?
A. "Do you have any joint pain?"
B. "Are you having any diarrhea?"
C. "Do you have frequent headaches?"
D. "Are you experiencing heartburn?"
Answer: A. "Do you have any joint pain?"
Rationale:
Leflunomide is an immunosuppressive agent and has an anti-inflammatory action. The
medication provides symptomatic relief of rheumatoid arthritis. Diarrhea can occur as a side
effect of the medication. The other options are unrelated to medication effectiveness.
174. A client with portosystemic encephalopathy is receiving oral lactulose (Chronulac) daily.
The nurse assesses which of the following to determine medication effectiveness?
A. Lung sounds
B. Blood pressure
C. Blood ammonia level
D. Serum potassium level
Answer: C. Blood ammonia level
Rationale:
Lactulose is a hyperosmotic laxative and ammonia detoxicant. It is used to prevent or treat
portosystemic encephalopathy, including hepatic precoma and coma. It also is used to treat
constipation. The medication retains ammonia in the colon (decreases the blood ammonia
concentration), producing an osmotic effect. It promotes increased peristalsis and bowel
evacuation, expelling ammonia from the colon.
175. A nurse notes that a client is receiving lamivudine (Epivir). The nurse determines that
this medication has been prescribed to treat which of the following?

A. Pancreatitis
B. Pharyngitis
C. Tonic-clonic seizures
D. Human immunodeficiency virus (HIV) infection
Answer: D. Human immunodeficiency virus (HIV) infection
Rationale:
Lamivudine is a nucleoside reverse transcriptase inhibitor and antiviral medication. It slows
HIV replication and reduces the progression of HIV infection. It also is used to treat chronic
hepatitis B and is used for prophylaxis in health care workers at risk of acquiring HIV after
occupational exposure to the virus. *Note the letters "-vir" in the trade name for this
medication *
176. A nurse notes that a client is taking lansoprazole (Prevacid). On data collection, the
nurse asks which question to determine medication effectiveness?
A. "Has your appetite increased?"
B. "Are you experiencing any heartburn?"
C. "Do you have any problems with vision?"
D. "Do you experience any leg pain when walking?"
Answer: B. "Are you experiencing any heartburn?"
Rationale:
Lansoprazole is a gastric acid pump inhibitor used to treat gastric and duodenal ulcers,
erosive esophagitis, and hypersecretory conditions. It also is used to treat gastroesophageal
reflux disease (GERD). It is not used to treat visual problems, problems with appetite, or leg
pain. *NOTE: "-zole" refers to gastric acid pump inhibitors*
177. A nurse is assisting in caring for a pregnant client who is receiving intravenous
magnesium sulfate for the management of preeclampsia and notes that the client's deep
tendon reflexes are absent. On the basis of this data, the nurse reports the finding and makes
which determination?
A. The magnesium sulfate is effective.
B. The infusion rate needs to be increased.
C. The client is experiencing cerebral edema.
D. The client is experiencing magnesium toxicity.
Answer: D. The client is experiencing magnesium toxicity.

Rationale:
Magnesium toxicity can occur as a result of magnesium sulfate therapy. Signs of magnesium
sulfate toxicity relate to the central nervous system depressant effects of the medication and
include respiratory depression; loss of deep tendon reflexes; sudden decrease in fetal heart
rate or maternal heart rate, or both; and sudden drop in blood pressure. Hyperreflexia
indicates increased cerebral edema. An absence of reflexes indicates magnesium toxicity. The
therapeutic serum level of magnesium for a client receiving magnesium sulfate ranges from 4
to 7.5 mEq/L (5 to 8 mg/dL).
178. Methylergonovine (Methergine) is prescribed for a client with postpartum hemorrhage
caused by uterine atony. Before administering the medication, the nurse checks which of the
following as the important client parameter?
A. Temperature
B. Lochial flow
C. Urine output
D. Blood pressure
Answer: D. Blood Pressure
Rationale:
Methylergonovine is an ergot alkaloid used for postpartum hemorrhage. It stimulates
contraction of the uterus and causes arterial vasoconstriction. Ergot alkaloids are avoided in
clients with significant cardiovascular disease, peripheral disease, hypertension, eclampsia, or
preeclampsia. These conditions are worsened by the vasoconstrictive effects of the ergot
alkaloids. The nurse would check the client's blood pressure before administering the
medication and would follow agency protocols regarding withholding of the medication.
Options 1, 2, and 3 are items that are checked in the postpartum period, but they are unrelated
to the use of this medication.
179. A nurse provides medication instructions to a client who had a kidney transplant about
therapy with cyclosporine (Sandimmune). Which statement by the client indicates a need for
further instruction?
A. "I need to obtain a yearly influenza vaccine."
B. "I need to have dental checkups every 3 months."
C. "I need to self-monitor my blood pressure at home."

D. "I need to call the health care provider (HCP) if my urine volume decreases or my urine
becomes cloudy."
Answer: A. "I need to obtain a yearly influenza vaccine."
Rationale:
Cyclosporine is an immunosuppressant medication. Because of the medication's effects, the
client should not receive any vaccinations without first consulting the HCP. The client should
report decreased urine output or cloudy urine, which could indicate kidney rejection or
infection, respectively. The client must be able to self-monitor blood pressure to check for the
side effect of hypertension. The client needs meticulous oral care and dental cleaning every 3
months to help prevent gingival hyperplasia.
180. A health care provider (HCP) writes a prescription for digoxin (Lanoxin), 0.25 mg daily.
The nurse teaches the client about the medication and tells the client that it is important to:
A. Count the radial and carotid pulses every morning.
B. Check the blood pressure every morning and evening.
C. Stop taking the medication if the pulse is higher than 100 beats per minute.
D. Withhold the medication and call the HCP if the pulse is less than 60 beats per minute. 1.
Answer: D. Withhold the medication and call the HCP if the pulse is less than 60 beats per
minute.
Rationale:
An important component of taking this medication is monitoring the pulse rate; however, it is
not necessary for the client to take both the radial and carotid pulses. It is not necessary for
the client to check the blood pressure every morning and evening because the medication
does not directly affect blood pressure. It is most important for the client to know the
guidelines related to withholding the medication and calling the HCP. The client should not
stop taking a medication.
181. A client is taking ticlopidine hydrochloride (Ticlid). The nurse tells the client to avoid
which of the following while taking this medication?
A. Vitamin C
B. Vitamin D
C. Acetaminophen (Tylenol)
D. Acetylsalicylic acid (aspirin)
Answer: D. Acetylsalicylic acid (aspirin)

Rationale:
Ticlopidine hydrochloride is a platelet aggregation inhibitor. It is used to decrease the risk of
thrombotic strokes in clients with precursor symptoms. Because it is an antiplatelet agent,
other medications that precipitate or aggravate bleeding should be avoided during its use.
Therefore, aspirin or any aspirin-containing product should be avoided.
182. A client with angina pectoris is experiencing chest pain that radiates down the left arm.
The nurse administers a sublingual nitroglycerin tablet to the client. The client's pain is
unrelieved, and the nurse determines that the client needs another nitroglycerin tablet. Which
of the following vital signs is most important for the nurse to check before administering the
medication?
A. Temperature
B. Respirations
C. Blood pressure
D. Radial pulse rate
Answer: C. Blood pressure
Rationale:
Nitroglycerin acts directly on the smooth muscle of the blood vessels, causing relaxation and
dilation. As a result, hypotension can occur. The nurse would check the client's blood
pressure before administering the second nitroglycerin tablet. Although the respirations and
apical pulse may be checked, these vital signs are not affected as a result of this medication.
The temperature also is not associated with the administration of this medication.
183. A client who received a kidney transplant is taking azathioprine (Imuran), and the nurse
provides instructions about the medication. Which statement by the client indicates a need for
further instructions?
A. "I need to watch for signs of infection."
B. "I need to discontinue the medication after 14 days of use."
C. "I can take the medication with meals to minimize nausea."
D. "I need to call the health care provider (HCP) if more than one dose is missed."
Answer: B. "I need to discontinue the medication after 14 days of use."
Rationale:
Azathioprine is an immunosuppressant medication that is taken for life. Because of the
effects of the medication, the client must watch for signs of infection, which are reported

immediately to the HCP. The client should also call the HCP if more than one dose is missed.
The medication may be taken with meals to minimize nausea.
184. A nurse preparing a client for surgery reviews the client's medication record. The client
is to be nothing per mouth (NPO) after midnight. Which of the following medications, if
noted on the client's record, should the nurse question?
A. Cyclobenzaprine (Flexeril)
B. Alendronate (Fosamax)
C. Allopurinol (Zyloprim)
D. Prednisone
Answer: D. Prednisone
Rationale:
Prednisone is a corticosteroid that can cause adrenal atrophy, which reduces the body's ability
to withstand stress. Before and during surgery, dosages may be temporarily increased.
Cyclobenzaprine is a skeletal muscle relaxant. Alendronate is a bone-resorption inhibitor.
Allopurinol is an antigout medication.
185. Which of the following herbal therapies would be prescribed for its use as an
antispasmodic? Select all that apply.
A. Aloe
B. Kava
C. Ginger
D. Chamomile
E. Peppermint oil
Answer: D. Chamomile
E. Peppermint oil
Rationale:
Chamomile has a mild sedative effect and acts as an antispasmodic and anti-inflammatory.
Peppermint oil acts as an antispasmodic and is used for irritable bowel syndrome. Topical
aloe promotes wound healing. Aloe taken orally acts as a laxative. Kava has an anxiolytic,
sedative, and analgesic effect. Ginger is effective in relieving nausea.

186. A nurse prepares to administer sodium polystyrene sulfonate (Kayexalate) to a client.
Before administering the medication, the nurse reviews the action of the medication and
understands that it:
A. Releases bicarbonate in exchange for primarily sodium ions
B. Releases sodium ions in exchange for primarily potassium ions
C. Releases potassium ions in exchange for primarily sodium ions
D. Releases sodium ions in exchange for primarily bicarbonate ions
Answer: B. Releases sodium ions in exchange for primarily potassium ions
Rationale:
Sodium polystyrene sulfonate is a cation exchange resin used in the treatment of
hyperkalemia. The resin either passes through the intestine or is retained in the colon. It
releases sodium ions in exchange for primarily potassium ions. The therapeutic effect occurs
2 to 12 hours after oral administration and longer after rectal administration.
187. A clinic nurse prepares to administer an MMR (measles, mumps, rubella) vaccine to a
child. How is this vaccine best administered?
A. Intramuscularly in the deltoid muscle
B. Subcutaneously in the gluteal muscle
C. Subcutaneously in the outer aspect of the upper arm
D. Intramuscularly in the anterolateral aspect of the thigh
Answer: C. Subcutaneously in the outer aspect of the upper arm
Rationale:
The MMR vaccine is administered subcutaneously in the outer aspect of the upper arm. The
gluteal muscle is most often used for intramuscular injections. The MMR vaccine is not
administered by the intramuscular route.
188. The nurse should anticipate that the most likely medication to be prescribed
prophylactically for a child with spina bifida (myelomeningocele) who has a neurogenic
bladder would be:
A. Prednisone
B. Sulfisoxazole
C. Furosemide (Lasix)
D. Intravenous immune globulin (IVIG)
Answer: B. Sulfisoxazole

Rationale:
A neurogenic bladder prevents the bladder from completely emptying because of the decrease
in muscle tone. The most likely medication to be prescribed to prevent urinary tract infection
would be an antibiotic. A common prescribed medication is sulfisoxazole. Prednisone
relieves allergic reactions and inflammation rather than preventing infection. Furosemide
promotes diuresis and decreases edema caused by congestive heart failure. IVIG assists with
antibody production in immunocompromised clients.
189. Prostaglandin E1 is prescribed for a child with transposition of the great arteries. The
mother of the child asks the nurse why the child needs the medication. The nurse tells the
mother that the medication:
A. Prevents hypercyanotic (blue or tet) spells
B. Maintains an adequate hormone level
C. Maintains the position of the great arteries
D. Provides adequate oxygen saturation and maintains cardiac output
Answer: D. Provides adequate oxygen saturation and maintains cardiac output
Rationale:
A child with transposition of the great arteries may receive prostaglandin E1 temporarily to
increase blood mixing if systemic and pulmonary mixing are inadequate to maintain adequate
cardiac output. Options 1, 2, and 3 are incorrect. In addition, hypercyanotic spells occur in
tetralogy of Fallot. *Use the ABCs—airway, breathing, and circulation—to answer the
question. The correct option addresses circulation*
190. A child is hospitalized with a diagnosis of lead poisoning. The nurse assisting in caring
for the child would prepare to assist in administering which of the following medications?
A. Activated charcoal
B. Sodium bicarbonate
C. Syrup of ipecac syrup
D. Dimercaprol (BAL in Oil)
Answer: D. Dimercaprol(BALin Oil)
Rationale:
Dimercaprol is a chelating agent that is administered to remove lead from the circulating
blood and from some tissues and organs for excretion in the urine. Sodium bicarbonate may
be used in salicylate poisoning. Syrup of ipecac is used in the hospital setting in poisonings to

induce vomiting. Activated charcoal is used to decrease absorption in certain poisoning
situations. Note that dimercaprol is prepared with peanut oil, and hence should be avoided by
clients with known or suspected peanut allergy.
191. A child is brought to the emergency department for treatment of an acute asthma attack.
The nurse prepares to administer which of the following medications first?
A. Oral corticosteroids
B. A leukotriene modifier
C. A β2 agonist
D. A nonsteroidal anti- inflammatory
Answer: C. A β2 agonist
Rationale:
In treating an acute asthma attack, a short acting β2 agonist such as albuterol (Proventil HFA)
will be given to produce bronchodilation. Options 1, 2, and 4 are long-term control
(preventive) medications.
192. A nurse is collecting medication information from a client, and the client states that she
is taking garlic as an herbal supplement. The nurse understands that the client is most likely
treating which of the following conditions?
A. Eczema
B. Insomnia
C. Migraines
D. Hyperlipidemia
Answer: D. Hyperlipidemia
Rationale:
Garlic is an herbal supplement that is used to treat hyperlipidemia and hypertension. An
herbal supplement that may be used to treat eczema is evening primrose. Insomnia has been
treated with both valerian root and chamomile. Migraines have been treated with feverfew.
193. Sodium hypochlorite (Dakin's solution) is prescribed for a client with a leg wound
containing purulent drainage. The nurse is assisting in developing a plan of care for the client
and includes which of the following in the plan?
A. Ensure that the solution is freshly prepared before use.
B. Soak a sterile dressing with solution and pack into the wound.

C. Allow the solution to remain in the wound following irrigation.
D. Apply the solution to the wound and on normal skin tissue surrounding the wound.
Answer: A. Ensure that the solution is freshly prepared before use.
Rationale:
Dakin solution is a chloride solution that is used for irrigating and cleaning necrotic or
purulent wounds. It can be used for packing necrotic wounds. It cannot be used to pack
purulent wounds because the solution is inactivated by copious pus. It should not come into
contact with healing or normal tissue, and it should be rinsed off immediately if used for
irrigation. Solutions are unstable and the nurse must ensure that the solution has been
prepared fresh before use. *Eliminate options 2 and 3 first because they are comparable or
alike. It makes sense to ensure that the solution is freshly prepared; therefore, select option 1*
194. A nurse provides instructions to a client regarding the use of tretinoin (Retin-A). Which
statement by the client indicates the need for further instructions?
A. "Optimal results will be seen after 6 weeks."
B. "I should apply a very thin layer to my skin."
C. "I should wash my hands thoroughly after applying the medication."
D. "I should cleanse my skin thoroughly before applying the medication."
Answer: B. I should apply a very thin layer to my skin.
Rationale:
Tretinoin is applied liberally to the skin. The hands are washed thoroughly immediately after
applying. Therapeutic results should be seen after 2 to 3 weeks but may not be optimal until
after 6 weeks. The skin needs to be cleansed thoroughly before applying the medication.
195. A nurse is caring for a client who is taking metoprolol (Lopressor). The nurse measures
the client's blood pressure (BP) and apical pulse (AP) immediately before administration. The
client's BP is following is the appropriate action?
A. Withhold the medication.
B. Notify the registered nurse immediately.
C. Administer the medication as prescribed.
D. Administer half of the prescribed medication.
Answer: A. Withhold the medication.
Rationale:

Metoprolol (Lopressor) is classified as a beta-adrenergic blocker and is used in the treatment
of hypertension, angina, and myocardial infarction. Baseline nursing assessments include
measurement of BP and AP immediately before administration. If the systolic BP is below 90
mm/Hg and the AP is below 60 beats/min, the nurse should withhold the medication and
document this action. Although the registered nurse should be informed of the client's vital
signs, it is not necessary to do so immediately. The medication should not be administered
because the data is outside of the prescribed parameters for this medication. The nurse should
not administer half of the medication, or alter any dosages at any point in time.
196. A client has been prescribed amikacin (Amikin). Which of the following priority
baseline functions should be monitored?
A. Apical pulse
B. Liver function
C. Blood pressure
D. Hearing acuity
Answer: D. Hearing acuity
Rationale:
Amikacin (Amikin) is an antibiotic. This medication can cause ototoxicity and
nephrotoxicity; therefore, hearing acuity tests and kidney function studies should be
performed before the initiation of therapy. Apical pulse, liver function studies, and blood
pressure are not specifically related to the use of this medication.
197. Collagenase (Santyl) is prescribed for a client with a severe burn to the hand. The nurse
provides instructions to the client regarding the use of the medication. Which statement by
the client indicates an accurate understanding of the use of this medication?
A. "I will apply the ointment once a day and leave it open to the air."
B. "I will apply the ointment twice a day and leave it open to the air."
C. "I will apply the ointment once a day and cover it with a sterile dressing."
D. "I will apply the ointment at bedtime and in the morning and cover it with a sterile
dressing."
Answer: C. "I will apply the ointment once a day and cover it with a sterile dressing."
Rationale:
Collagenase is used to promote debridement of dermal lesions and severe burns. It is usually
applied once daily and covered with a sterile dressing.

198. Coal tar has been prescribed for a client with a diagnosis of psoriasis, and the nurse
provides instructions to the client about the medication. Which statement by the client
indicates a need for further instructions?
A. "The medication can cause phototoxicity."
B. "The medication has an unpleasant odor."
C. "The medication can stain the skin and hair."
D. "The medication can cause systemic effects."
Answer: D. "The medication can cause systemic effects."
Rationale:
Coal tar is used to treat psoriasis and other chronic disorders of the skin. It suppresses DNA
synthesis, mitotic activity, and cell proliferation. It has an unpleasant odor, can frequently
stain the skin and hair, and can cause phototoxicity. Systemic toxicity does not occur. *The
name of the medication will assist in eliminating options 2 and 3*
199. A nurse is applying a topical glucocorticoid to a client with eczema. The nurse monitors
for systemic absorption of the medication if the medication is being applied to which of the
following body areas?
A. Back
B. Axilla
C. Soles of the feet
D. Palms of the hands
Answer: B. Axilla
Rationale:
Topical glucocorticoids can be absorbed into the systemic circulation. Absorption is higher
from regions where the skin is especially permeable (scalp, axillae, face, eyelids, neck,
perineum, genitalia), and lower from regions where penetrability is poor (back, palms, soles).
*Eliminate options 3 and 4 because these body areas are similar in terms of skin
characteristics*
200. A client is seen in the clinic for complaints of skin itchiness that has been persistent over
the past several weeks. Following data collection, it has been determined that the client has
scabies. Lindane is prescribed, and the nurse is asked to provide instructions to the client
regarding the use of the medication. The nurse tells the client to:

A. Apply a thick layer of cream to the entire body.
B. Apply the cream as prescribed for 2 days in a row.
C. Apply to the entire body and scalp, excluding the face.
D. Leave the cream on for 8 to 12 hours and then remove by washing.
Answer: D. Leave the cream on for 8 to 12 hours and then remove by washing.
Rationale:
Lindane is applied in a thin layer to the entire body below the head. No more than 30 g (1 oz)
should be used. The medication is removed by washing 8 to 12 hours later. Usually, only one
application is required.
201. A nurse is preparing to administer eardrops to an infant. The nurse plans to:
A. Pull up and back on the ear and direct the solution onto the eardrum.
B. Pull down and back on the ear and direct the solution onto the eardrum.
C. Pull down and back on the ear and direct the solution toward the wall of the canal.
D. Pull up and back on the ear lobe and direct the solution toward the wall of the canal. 2.
Answer: C. Pull down and back on the ear and direct the solution toward the wall of the
canal.
Rationale:
When administering eardrops to an infant, the nurse pulls the ear down and straight back. In
the adult or a child older than 3 years, the ear is pulled up and back to straighten the auditory
canal. The medication is administered by aiming it at the wall of the canal rather than directly
onto the eardrum.
202. A nurse is collecting data from a client about medications being taken, and the client
tells the nurse that he is taking herbal supplements for the treatment of varicose veins. The
nurse understands that the client is most likely taking which of the following?
A. Bilberry
B. Ginseng
C. Feverfew
D. Evening primrose
Answer: A. Bilberry
Rationale:
Bilberry is an herbal supplement that has been used to treat varicose veins. This supplement
has also been used to treat cataracts, retinopathy, diabetes mellitus, and peripheral vascular

disease. Ginseng has been used to improve memory performance and decrease blood glucose
levels in type 2 diabetes mellitus. Feverfew is used to prevent migraine headaches and to treat
rheumatoid arthritis. Evening primrose is used to treat eczema and skin irritation.
203. A nurse is preparing to give the postcraniotomy client medication for incisional pain.
The family asks the nurse why the client is receiving codeine sulfate and not "something
stronger." In formulating a response, the nurse incorporates the understanding that codeine:
A. Is one of the strongest opioid analgesics available
B. Cannot lead to physical or psychological dependence
C. Does not cause gastrointestinal upset or constipation as do other opioids
D. Does not alter respirations or mask neurological signs as do other opioids
Answer: D. Does not alter respirations or mask neurological signs as do other opioids
Rationale:
Codeine sulfate is the opioid analgesic often used for clients after craniotomy. It is frequently
combined with a nonopioid analgesic such as acetaminophen for added effect. It does not
alter the respiratory rate or mask neurological signs as do other opioids. Side effects of
codeine include gastrointestinal upset and constipation. The medication can lead to physical
and psychological dependence with chronic use. It is not the strongest opioid analgesic
available.
204. A client receives a dose of edrophonium (Enlon). The client shows improvement in
muscle strength for a period of time following the injection. The nurse interprets that this
finding is compatible with:
A. Multiple sclerosis
B. Myasthenia gravis
C. Muscular dystrophy
D. Amyotrophic lateral sclerosis
Answer: B. Myasthenia gravis
Rationale:
Myasthenia gravis can often be diagnosed based on clinical signs and symptoms. The
diagnosis can be confirmed by injecting the client with a dose of edrophonium . This
medication inhibits the breakdown of an enzyme in the neuromuscular junction, so more
acetylcholine binds to receptors. If the muscle is strengthened for 3 to 5 minutes after this
injection, it confirms a diagnosis of myasthenia gravis. Another medication, neostigmine

(Prostigmin), also may be used because its effect lasts for 1 to 2 hours, providing a better
analysis. For either medication, atropine sulfate should be available as the antidote.
205. A nurse is assisting in preparing to administer acetylcysteine (Mucomyst) to a client with
an overdose of acetaminophen (Tylenol). The nurse prepares to administer the medication by:
A. Administering the medication subcutaneously in the deltoid muscle
B. Administering the medication by the intramuscular route in the gluteal muscle
C. Administering the medication by the intramuscular route, mixed in 10 mL of normal saline
D. Mixing the medication in a flavored ice drink and allowing the client to drink the
medication through a straw
Answer: D. Mixing the medication in a flavored ice drink and allowing the client to drink the
medication through a straw
Rationale:
Because acetylcysteine has a pervasive odor of rotten eggs, it must be disguised in a flavored
ice drink. It is consumed preferably through a straw to minimize contact with the mouth. It is
not administered by the intramuscular or subcutaneous route. *Knowing that the medication
is a solution that is also used for nebulization treatments will assist you to select the option
that indicates an oral route*
206. A client is receiving baclofen (Lioresal) for muscle spasms caused by a spinal cord
injury. The nurse monitors the client, knowing that which of the following is a side effect of
this medication?
A. Muscle pain
B. Hypertension
C. Slurred speech
D. Photosensitivity
Answer: C. Slurred speech
Rationale:
Side effects of baclofen include drowsiness, dizziness, weakness, and nausea. Occasional side
effects include headache, paresthesia of the hands and feet, constipation or diarrhea, anorexia,
hypotension, confusion, and nasal congestion. Paradoxical central nervous system excitement
and restlessness can occur, along with slurred speech, tremor, dry mouth, nocturia, and
impotence. *Option 3 is most closely associated with a neurological disorder*

207. A client is suspected of having myasthenia gravis, and the health care provider
administers edrophonium (Enlon) to determine the diagnosis. After administration of this
medication, which of the following would indicate the presence of myasthenia gravis?
A. Joint pain
B. A decrease in muscle strength
C. An increase in muscle strength
D. Feelings of faintness, dizziness, hypotension, and signs of flushing in the client
Answer: C. An increase in muscle strength
Rationale:
Edrophonium is a short-acting acetylcholinesterase inhibitor used as a diagnostic agent.
When a client with suspected myasthenia gravis is given the medication intravenously, an
increase in muscle strength would be seen in 1 to 3 minutes. If no response occurs, another
dose is given over the next 2 minutes, and muscle strength is again tested. If no increase in
muscle strength occurs with this higher dose, the muscle weakness is not caused by
myasthenia gravis. Clients receiving injections of this medication commonly demonstrate a
drop of blood pressure, feel faint and dizzy, and are flushed.
208. A client with myasthenia gravis verbalizes complaints of feeling much weaker than
normal. The health care provider plans to implement a diagnostic test to determine if the
client is experiencing a myasthenic crisis and administers edrophonium (Enlon). Which of the
following would indicate that the client is experiencing a myasthenic crisis?
A. Increasing weakness
B. No change in the condition
C. An increase in muscle spasms
D. A temporary improvement in the condition
Answer: D. A temporary improvement in the condition
Rationale:
Edrophonium (Enlon) is administered to determine whether the client is reacting to an
overdose of a medication (cholinergic crisis) or to an increasing severity of the disease
(myasthenic crisis). When the edrophonium (Enlon) injection is given and the condition
improves temporarily, the client is in myasthenic crisis. This is known as a positive test.
Increasing weakness would occur in cholinergic crisis. Options 2 and 3 would not occur in
either crisis.

209. A client with multiple sclerosis is receiving diazepam (Valium), a centrally acting
skeletal muscle relaxant. Which of the following would indicate that the client is
experiencing a side effect related to this medication?
A. Headache
B. Drowsiness
C. Urinary retention
D. Increased salivation
Answer: B. Drowsiness
Rationale:
Incoordination and drowsiness are common side effects resulting from this medication.
Options 1, 3, and 4 are incorrect.
210. Dantrolene (Dantrium) is prescribed for a client with a spinal cord injury for discomfort
resulting from spasticity. The nurse tells the client about the importance of follow-up and the
need for which blood study?
A. Creatinine level
B. Sedimentation rate
C. Liver function studies
D. White blood cell count
Answer: C. Liver function studies
Rationale:
Dantrolene can cause liver damage, and the nurse should monitor liver function studies.
Baseline liver function studies are done before therapy starts, and regular liver function
studies are performed throughout therapy. Dantrolene is discontinued if no relief of spasticity
is achieved in 6 weeks.
211. A client with epilepsy is taking the prescribed dose of phenytoin (Dilantin) to control
seizures. A phenytoin blood level is drawn, and the results reveal a level of 35 mcg/ml.
Which of the following symptoms would be expected as a result of this laboratory result?
A. Nystagmus
B. Tachycardia
C. Slurred speech
D. No symptoms, because this is a normal therapeutic level
Answer: C. Slurred speech

Rationale:
The therapeutic phenytoin level is 10 to 20 mcg/mL. At a level higher than 20 mcg/mL,
involuntary movements of the eyeballs (nystagmus) appear. At a level higher than 30
mcg/mL, ataxia and slurred speech occur.
212. Mannitol (Osmitrol) is being administered to a client with increased intracranial pressure
following a head injury. The nurse assisting in caring for the client knows that which of the
following indicates the therapeutic action of this medication?
A. Prevents the filtration of sodium and water through the kidneys
B. Prevents the filtration of sodium and potassium through the kidneys
C. Decreases water loss by promoting the reabsorption of sodium and water in the loop of
Henle
D. Induces diuresis by raising the osmotic pressure of glomerular filtrate, thereby inhibiting
tubular reabsorption of water and solutes
Answer: D. Induces diuresis by raising the osmotic pressure of glomerular filtrate, thereby
inhibiting tubular reabsorption of water and solutes
Rationale:
Mannitol is an osmotic diuretic that induces diuresis by raising the osmotic pressure of
glomerular filtrate, thereby inhibiting tubular reabsorption of water and solutes. It is used to
reduce intracranial pressure in the client with head trauma.
213. A client is admitted to the hospital with complaints of back spasms. The client states, "I
have been taking two or three aspirin every 4 hours for the past week and it hasn't helped my
back." Aspirin intoxication is suspected. Which of the following complaints would indicate
aspirin intoxication?
A. Tinnitus
B. Constipation
C. Photosensitivity
D. Abdominal cramps
Answer: A. Tinnitus
Rationale:
Mild intoxication with acetylsalicylic acid (aspirin) is called salicylism and is commonly
experienced when the daily dosage is higher than 4 g. Tinnitus (ringing in the ears) is the
most frequently occurring effect noted with intoxication. Hyperventilation may occur because

salicylate stimulates the respiratory center. Fever may result because salicylate interferes with
the metabolic pathways involved with oxygen consumption and heat production. Options 2,
3, and 4 are incorrect.
214. A health care provider initiates carbidopa/levodopa (Sinemet) therapy for the client with
Parkinson's disease. A few days after the client starts the medication, the client complains of
nausea and vomiting. The nurse tells the client that:
A. Taking an antiemetic is the best measure to prevent the nausea.
B. Taking the medication with food will help to prevent the nausea.
C. This is an expected side effect of the medication and will decrease over time.
D. The nausea and vomiting will decrease when the dose of levodopa is stabilized.
Answer: B. Taking the medication with food will help to prevent the nausea.
Rationale:
If carbidopa/levodopa is causing nausea and vomiting, the nurse would tell the client that
taking the medication with food will prevent the nausea. Additionally, the client should be
instructed not to take the medication with a high-protein meal because the high-protein will
affect absorption. Antiemetics from the phenothiazine class should not be used because they
block the therapeutic action of dopamine. *eliminate options 3 and 4 because they are
comparable or alike*
215. A client with rheumatoid arthritis is taking acetylsalicylic acid (aspirin) on a daily basis.
Which medication dose should the nurse expect the client to be taking?
A. 1 g daily
B. 4 g daily
C. 325 mg daily
D. 1000 mg daily
Answer: B. 4 g daily
Rationale:
Aspirin may be used to treat the client with rheumatoid arthritis. It may also be used to reduce
the risk of recurrent transient ischemic attack (TIA) or brain attack (stroke) or reduce the risk
of myocardial infarction (MI) in clients with unstable angina or a history of a previous MI.
The normal dose for clients being treated with aspirin to decrease thrombosis and MI is 300
to 325 mg/day. Clients being treated to prevent TIAs are usually prescribed 1.3 g/day in two

to four divided doses. Clients with rheumatoid arthritis are treated with 3.6 to 5.4 g/day in
divided doses. *Eliminate options 1 and 4 because they are alike*
216. A nurse is caring for a client with gout who is taking Colcrys (colchicine). The client has
been instructed to restrict the diet to low- purine foods. Which of the following foods should
the nurse instruct the client to avoid while taking this medication?
A. Spinach
B. Scallops
C. Potatoes
D. Ice cream
Answer: B. Scallops
Rationale:
Colchicine is a medication used for clients with gout to inhibit the reabsorption of uric acid
by the kidney and promote excretion of uric acid in the urine. Uric acid is produced when
purine is catabolized. Clients are instructed to modify their diet and limit excessive purine
intake. High-purine foods to avoid or limit include organ meats, roe, sardines, scallops,
anchovies, broth, mincemeat, herring, shrimp, mackerel, gravy, and yeast.
217. A health care provider prescribes auranofin (Ridaura) for a client with rheumatoid
arthritis. Which of the following would indicate to the nurse that the client is experiencing
toxicity related to the medication?
A. Joint pain
B. Constipation
C. Ringing in the ears
D. Complaints of a metallic taste in the mouth
Answer: D. Complaints of a metallic taste in the mouth
Rationale:
Ridaura is the one gold preparation that is given orally rather than by injection.
Gastrointestinal reactions including diarrhea, abdominal pain, nausea, and loss of appetite are
common early in therapy, but these usually subside in the first 3 months of therapy. Early
symptoms of toxicity include a rash, purple blotches, pruritus, mouth lesions, and a metallic
taste in the mouth.

218. A film-coated form of diflunisal has been prescribed for a client for the treatment of
chronic rheumatoid arthritis. The client calls the clinic nurse because of difficulty swallowing
the tablets. Which initial instruction should the nurse provide to the client?
A. "Crush the tablets and mix them with food."
B. "Notify the health care provider for a medication change."
C. "Open the tablet and mix the contents with food."
D. "Swallow the tablets with large amounts of water or milk."
Answer: D. "Swallow the tablets with large amounts of water or milk."
Rationale:
Diflunisal may be given with water, milk, or meals. The tablets should not be crushed or
broken open. Taking the medication with a large amount of water or milk should be tried
before contacting the health care provider.
219. A health care provider instructs a client with rheumatoid arthritis to take ibuprofen
(Motrin). The nurse reinforces the instructions, knowing that the normal adult dose for this
client is which of the following?
A. 100 mg orally twice a day
B. 200 mg orally twice a day
C. 400 mg orally three times a day
D. 1000 mg orally four times a day
Answer: C. 400 mg orally three times a day
Rationale:
For acute or chronic rheumatoid arthritis or osteoarthritis, the normal oral adult dose is 400 to
800 mg three or four times daily.
220. A adult client with muscle spasms is taking an oral maintenance dose of baclofen
(Lioresal). The nurse reviews the medication record, expecting that which dose should be
prescribed?
A. 15 mg four times a day
B. 25 mg four times a day
C. 30 mg four times a day
D. 40 mg four times a day
Answer: A. 15 mg four times a day
Rationale:

Baclofen is dispensed in 10- and 20-mg tablets for oral use. Dosages are low initially and
then gradually increased. Maintenance doses range from 15 to 20 mg administered three or
four times a day.
221. A nurse is reviewing the health care provider's prescriptions for an adult client who has
been admitted to the hospital following a back injury. Carisoprodol (Soma) is prescribed for
the client to relieve the muscle spasms; the health care provider has prescribed 350 mg to be
administered four times a day. When preparing to give this medication, the nurse determines
that this dosage is:
A. The normal adult dosage
B. A lower than normal dosage
C. A higher than normal dosage
D. A dosage requiring further clarification
Answer: A. The normal adult dosage
Rationale:
The normal adult dosage for carisoprodol is 350 mg orally three or four times daily.
222. A nurse has administered a dose of diazepam (Valium) to a client. The nurse would take
which important action before leaving the client's room?
A. Giving the client a bedpan
B. Drawing the shades or blinds closed
C. Turning down the volume on the television
D. Per agency policy, putting up the side rails on the bed
Answer: D. Per agency policy, putting up the side rails on the bed
Rationale:
Diazepam is a sedative-hypnotic with anticonvulsant and skeletal muscle relaxant properties.
The nurse should institute safety measures before leaving the client's room to ensure that the
client does not injure herself or himself. The most frequent side effects of this medication are
dizziness, drowsiness, and lethargy. For this reason, the nurse puts the side rails up on the bed
before leaving the room to prevent falls. Options 1, 2, and 3 may be helpful measures that
provide a comfortable, restful environment, but option 4 is the one that provides for the
client's safety needs.

223. A client with a psychotic disorder is being treated with haloperidol (Haldol). Which of
the following would indicate the presence of a toxic effect of this medication?
A. Nausea
B. Hypotension
C. Blurred vision
D. Excessive salivation
Answer: D. Excessive salivation
Rationale:
Toxic effects include extrapyramidal symptoms (EPS) noted as marked drowsiness and
lethargy, excessive salivation, and a fixed stare. Akathisia, acute dystonias, and tardive
dyskinesia are also signs of toxicity. Hypotension, nausea, and blurred vision are occasional
side effects.
224. Neuroleptic malignant syndrome is suspected in a client who is taking chlorpromazine.
Which medication would the nurse prepare in anticipation of being prescribed to treat this
adverse effect related to the use of chlorpromazine?
A. Protamine sulfate
B. Bromocriptine (Parlodel)
C. Phytonadione (vitamin K)
D. Enalapril maleate (Vasotec)
Answer: B. Bromocriptine (Parlodel)
Rationale:
Bromocriptine is an antiparkinsonian prolactin inhibitor used in the treatment of neuroleptic
malignant syndrome. Vitamin K is the antidote for warfarin (Coumadin) overdose. Protamine
sulfate is the antidote for heparin overdose. Enalapril maleate is an antihypertensive used in
the treatment of hypertension.
225. A nursing student is assigned to care for a client with a diagnosis of schizophrenia.
Haloperidol (Haldol) is prescribed for the client, and the nursing instructor asks the student to
describe the action of the medication. Which statement by the nursing student indicates an
understanding of the action of this medication?
A. It is a serotonin reuptake blocker.
B. It inhibits the breakdown of released acetylcholine.
C. It blocks the uptake of norepinephrine and serotonin.

D. It blocks the binding of dopamine to the postsynaptic dopamine receptors in the brain.
Answer: D. It blocks the binding of dopamine to the postsynaptic dopamine receptors in the
brain.
Rationale:
Haloperidol acts by blocking the binding of dopamine to the postsynaptic dopamine receptors
in the brain. Imipramine hydrochloride (Tofranil) blocks the reuptake of norepinephrine and
serotonin. Donepezil hydrochloride (Aricept) inhibits the breakdown of released
acetylcholine. Fluoxetine hydrochloride (Prozac) is a potent serotonin reuptake blocker.
226. A client receiving lithium carbonate (Lithobid) complains of loose, watery stools and
difficulty walking. The nurse would expect the serum lithium level to be which of the
following?
A. 0.7 mEq/L 2.
B. 1. 0 mEq/L 3.
C. 1. 2 mEq/L 4.
D. 1. 7 mEq/L
Answer: A. 7 mEq/L
Rationale:
The therapeutic serum level of lithium ranges from 0.6 to 1.2 mEq/L. Serum lithium levels
above the therapeutic level will produce signs of toxicity.
227. When teaching a client who is being started on imipramine hydrochloride (Tofranil), the
nurse would inform the client that the desired effects of the medication may:
A. Start during the first week of administration
B. Not occur for 2 to 3 weeks of administration
C. Start during the second week of administration
D. Not occur until after a month of administration
Answer: B. Not occur for 2 to 3 weeks of administration
Rationale:
The therapeutic effects of administration of imipramine hydrochloride may not occur for 2 to
3 weeks after the antidepressant therapy has been initiated. Therefore options 1, 3, and 4 are
incorrect.

228. A client receiving an anxiolytic medication complains that he feels very "faint" when he
tries to get out of bed in the morning. The nurse recognizes this complaint as a symptom of:
A. Cardiac dysrhythmias
B. Postural hypotension
C. Psychosomatic symptoms
D. Respiratory insufficiency
Answer: B. Postural hypotension
Rationale:
Anxiolytic medications can cause postural hypotension. The client needs to be taught to rise
to a sitting position and get out of bed slowly because of this adverse effect related to the
medication. Options 1, 3, and 4 are unrelated to the use of this medication.
229. A client who is taking lithium carbonate (Lithobid) is scheduled for surgery. The nurse
informs the client that:
A. The medication will be discontinued a week before the surgery and resumed 1 week
postoperatively.
B. The medication is to be taken until the day of surgery and resumed by injection
immediately postoperatively.
C. The medication will be discontinued 1 to 2 days before the surgery and resumed as soon as
full oral intake is allowed.
D. The medication will be discontinued several days before surgery and resumed by injection
in the immediate postoperative period.
Answer: C. The medication will be discontinued 1 to 2 days before the surgery and resumed
as soon as full oral intake is allowed.
Rationale:
The client who is on lithium carbonate must be off the medication for 1 to 2 days before a
scheduled surgical procedure and can resume the medication when full oral intake is
prescribed after the surgery. *lithium carbonate is an oral medication and is not given as an
injection*
230. A client is placed on chloral hydrate (Somnote) for short-term treatment. Which nursing
action indicates an understanding of the major side effect of this medication?
A. Monitoring neurological signs every 2 hours
B. Monitoring the blood pressure every 4 hours

C. Instructing the client to call for ambulation assistance
D. Lowering the bed and clearing a path to the bathroom at bedtime
Answer: C. Instructing the client to call for ambulation assistance
Rationale:
Chloral hydrate (a sedative-hypnotic) causes sedation and impairment of motor coordination;
therefore, safety measures need to be implemented. The client is instructed to call for
assistance with ambulation. Options 1 and 2 are not specifically associated with the use of
this medication. Although option 4 is an appropriate nursing intervention, it is most important
to instruct the client to call for assistance with ambulation.
231. A client admitted to the hospital gives the nurse a bottle of clomipramine (Anafranil).
The nurse notes that the medication has not been taken by the client in 2 months. What
behaviors observed in the client would validate noncompliance with this medication?
A. Complaints of hunger
B. Complaints of insomnia
C. A pulse rate less than 60 beats per minute
D. Frequent handwashing with hot, soapy water
Answer: D. Frequent handwashing with hot, soapy water
Rationale:
Clomipramine is commonly used in the treatment of obsessive-compulsive disorder.
Handwashing is a common obsessive-compulsive behavior. Weight gain is a common side
effect of this medication. Tachycardia and sedation are side effects. Insomnia may occur but
is seldom a side effect.
232. A client in the mental health unit is administered haloperidol (Haldol). The nurse would
check which of the following to determine medication effectiveness?
A. The client's vital signs
B. The client's nutritional intake
C. The physical safety of other unit clients
D. The client's orientation and delusional status
Answer: D. The client's orientation and delusional status
Rationale:
Haloperidol is used to treat clients exhibiting psychotic features. Therefore, to determine
medication effectiveness, the nurse would check the client's orientation and delusional status.

Vital signs are routine and not specific to this situation. The physical safety of other clients is
not a direct assessment of this client. Monitoring nutritional intake is not related to this
situation.
233. Diphenhydramine hydrochloride (Benadryl) is used in the treatment of allergic rhinitis
for a hospitalized client with a chronic psychotic disorder. The client asks the nurse why the
medication is being discontinued before hospital discharge. The nurse responds, knowing
that:
A. Allergic symptoms are short in duration.
B. This medication promotes long-term extrapyramidal symptoms.
C. Addictive properties are enhanced in the presence of psychotropic medications.
D. Poor compliance causes this medication to fail to reach its therapeutic blood level.
Answer: C. Addictive properties are enhanced in the presence of psychotropic medications.
Rationale:
The addictive properties of diphenhydramine hydrochloride are enhanced when used with
psychotropic medications. Allergic symptoms may not be short term and will occur if
allergens are present in the environment. Poor compliance may be a problem with psychotic
clients but is not the subject of the question. Diphenhydramine hydrochloride may be used for
extrapyramidal symptoms and mild medication-induced movement disorders.
234. A hospitalized client is started on phenelzine sulfate (Nardil) for the treatment of
depression. At lunchtime, a tray is delivered to the client. Which food item on the tray will
the nurse remove?
A. Yogurt
B. Crackers
C. Tossed salad
D. Oatmeal cookies
Answer: A. Yogurt
Rationale:
Phenelzine sulfate is a monoamine oxidase inhibitor (MAOI). The client should avoid taking
in foods that are high in tyramine. These foods could trigger a potentially fatal hypertensive
crisis. Foods to avoid include yogurt, aged cheeses, smoked or processed meats, red wines,
and fruits such as avocados, raisins, or figs.

235. A tricyclic antidepressant is administered to a client daily. The nurse plans to monitor for
the common side effects of the medication and includes which of the following in the plan of
care?
A. Offer hard candy or gum periodically.
B. Offer a nutritious snack between meals.
C. Monitor the blood pressure every 2 hours.
D. Review the white blood cell (WBC) count results daily.
Answer: A. Offer hard candy or gum periodically.
Rationale:
Dry mouth is a common side effect of tricyclic antidepressants. Frequent mouth rinsing with
water, sucking on hard candy, and chewing gum will alleviate this common side effect. It is
not necessary to monitor the blood pressure every 2 hours. In addition, it is not necessary to
check the WBC daily. Weight gain is a common side effect and frequent snacks will
aggravate this problem.
236. A client is being treated for depression with amitriptyline hydrochloride. During the
initial phases of treatment, the most important nursing intervention is:
A. Prescribing the client a tyramine- free diet
B. Checking the client for anticholinergic effects
C. Monitoring blood levels frequently because there is a narrow range between therapeutic
and toxic blood levels of this medication
D. Getting baseline postural blood pressures before administering the medication and each
time the medication is administered
Answer: D. Getting baseline postural blood pressures before administering the medication
and each time the medication is administered
Rationale:
Amitriptyline hydrochloride is a tricyclic antidepressant often used to treat depression. It
causes orthostatic changes and can produce hypotension and tachycardia. This can be
frightening to the client and dangerous because it can result in dizziness and client falls. The
client must be instructed to move slowly from a lying to a sitting to a standing position to
avoid injury if these effects are experienced. The client may also experience sedation, dry
mouth, constipation, blurred vision, and other anticholinergic effects, but these are transient
and will diminish with time.

237. A client who is on lithium carbonate (Lithobid) will be discharged at the end of the
week. In formulating a discharge teaching plan, the nurse will instruct the client that it is most
important to:
A. Avoid soy sauce, wine, and aged cheese.
B. Have the lithium level checked every week.
C. Take medication only as prescribed because it can become addicting.
D. Check with the psychiatrist before using any over-the- counter (OTC) medications or
prescription medications.
Answer: D. Check with the psychiatrist before using any over-the-counter (OTC)
medications or prescription medications.
Rationale:
Lithium is the medication of choice to treat manic-depressive illness. Many OTC medications
interact with lithium, and the client is instructed to avoid OTC medications while taking
lithium. Lithium is not addicting, and, although serum lithium levels need to be monitored, it
is not necessary to check these levels every week. A tyramine-free diet is associated with
monoamine oxidase inhibitors.
238. Ribavirin (Virazole) is prescribed for the hospitalized child with respiratory syncytial
virus (RSV). The nurse prepares to administer this medication via which of the following
routes?
A. Orally
B. Via face mask
C. Intravenously
D. Intramuscularly
Answer: B. Via face mask
Rationale:
Ribavirin is an antiviral respiratory medication used mainly in hospitalized children with
severe RSV and in high-risk children. Administration is via hood, face mask, or oxygen tent.
The medication is most effective if administered within the first 3 days of the infection.
239. Which of the following precautions will the nurse specifically take during the
administration of ribavirin (Virazole) to a child with respiratory syncytial virus (RSV)?
A. Wearing goggles
B. Wearing a gown

C. Wearing a gown and a mask
D. Handwashing before administration
Answer: A. Wearing goggles
Rationale:
Some caregivers experience headaches, burning nasal passages and eyes, and crystallization
of soft contact lenses as a result of administration of ribavirin. Specific to this medication is
the use of goggles. A gown is not necessary. A mask may be worn. Handwashing is to be
performed before and after any child contact.
240. A client with Parkinson's disease has been prescribed benztropine (Cogentin). The nurse
monitors for which gastrointestinal (GI) side effect of this medication?
A. Diarrhea
B. Dry mouth
C. Increased appetite
D. Hyperactive bowel sounds
Answer: B. Dry mouth
Rationale:
Common GI side effects of benztropine therapy include constipation and dry mouth. Other GI
side effects include nausea and ileus. These effects are the result of the anticholinergic
properties of the medication. *Eliminate options 1 and 4 because they are comparable or
alike. Recall that the medication is an anticholinergic, which causes dry mouth*
241. A client with a history of simple partial seizures is taking clorazepate (Tranxene), and
asks the nurse if there is a risk of addiction. The nurse's response is based on the
understanding that clorazepate:
A. Is not habit forming, either physically or psychologically
B. Leads to physical tolerance, but only after 10 or more years of therapy
C. Leads to physical and psychological dependence with prolonged high-dose therapy
D. Can result in psychological dependence only, because of the nature of the medication
Answer: C. Leads to physical and psychological dependence with prolonged high-dose
therapy
Rationale:
Clorazepate is classified as an anticonvulsant, antianxiety agent, and sedative-hypnotic
(benzodiazepine). One of the concerns with clorazepate therapy is that the medication can

lead to physical or psychological dependence with prolonged therapy at high doses. For this
reason, the amount of medication that is readily available to the client at any one time is
restricted. *Eliminate options 2 and 4 first because of the closed- ended word "only"*
242. A client who was started on anticonvulsant therapy with clonazepam (Klonopin) tells the
nurse of increasing clumsiness and unsteadiness since starting the medication. The client is
visibly upset by these manifestations and asks the nurse what to do. The nurse's response is
based on the understanding that these symptoms:
A. Usually occur if the client takes the medication with food
B. Are probably the result of an interaction with another medication
C. Indicate that the client is experiencing a severe untoward reaction to the medication
D. Are worse during initial therapy and decrease or disappear with long-term use
Answer: D. Are worse during initial therapy and decrease or disappear with long-term use
Rationale:
Drowsiness, unsteadiness, and clumsiness are expected effects of the medication during early
therapy. They are dose related and usually diminish or disappear altogether with continued
use of the medication. It does not indicate that a severe side effect is occurring. It is also
unrelated to interaction with another medication. The client is encouraged to take this
medication with food to minimize gastrointestinal upset. *Eliminate options 2 and 3 first
because they are comparable or alike and because of the word "severe" in option 3*
243. A hospitalized client is having the dosage of clonazepam (Klonopin) adjusted. The nurse
should plan to:
A. Weigh the client daily.
B. Observe for ecchymosis.
C. Institute seizure precautions.
D. Monitor blood glucose levels.
Answer: C. Institute seizure precautions.
Rationale:
Clonazepam is a benzodiazepine used as an anticonvulsant. During initial therapy and during
periods of dosage adjustment, the nurse should initiate seizure precautions for the client.
Options 1, 2, and 4 are not associated with the use of this medication.

244. A client has a prescription for valproic acid (Depakene) orally once daily. The nurse
plans to:
A. Administer the medication with an antacid.
B. Administer the medication with a carbonated beverage.
C. Ensure that the medication is administered at the same time each day.
D. Ensure that the medication is administered 2 hours before breakfast only, when the client's
stomach is empty.
Answer: C. Ensure that the medication is administered at the same time each day.
Rationale:
Valproic acid is an anticonvulsant, antimanic, and antimigraine medication. It may be
administered with or without food. It should not be taken with an antacid or carbonated
beverage because these products will affect medication absorption. The medication is
administered at the same time each day to maintain therapeutic serum levels. *Use general
pharmacology guidelines to assist in eliminating options 1 and 2. Eliminate option 4 because
of the closed-ended word "only."*
245. A client taking carbamazepine (Tegretol) asks the nurse what to do if he misses one
dose. The nurse responds that the carbamazepine should be:
A. Withheld until the next scheduled dose
B. Withheld and the health care provider is notified immediately
C. Taken as long as it is not immediately before the next dose
D. Withheld until the next scheduled dose, which should then be doubled
Answer: C. Taken as long as it is not immediately before the next dose
Rationale:
Carbamazepine is an anticonvulsant that should be taken around the clock, precisely as
directed. If a dose is omitted, the client should take the dose as soon as it is remembered, as
long as it is not immediately before the next dose. The medication should not be double
dosed. If more than one dose is omitted, the client should call the health care provider.
246. -afil
Answer: Erectile dysfunction s/s: headache, heartburn, diarrhea, flushing, nosebleeds,
parathesias, changes in color vision Contradicted in clients taking nitrates, anticoags, anti
HTN Common meds- sildenafil (viagra)

247. albuterol
Answer: Bronchodilator S/S: tachcardia, palpitations, tremors
248. alteplase (activase, tPA)
Answer: dissolves clots TX: acute MI, DVT, massive PE, ischemic stroke S/S: serious
bleeding risks from recent wounds, puncture sites, weakened vessels, hypotension NI: Must
take 4-6 hrs of onset
249. amitripytyline (elavil)
Answer: TCA S/S: anticholenergic effects, sedation, toxicity NI: DO NOT admin with
MAOIs, avoid alcohol, contradicted in clients w/ seizures
250. -arin
Answer: Anticoagulant inhibit clotting factors (warfarin = factors VII, IX, X) TX: evolving
stroke, pulmonary embolism, massive deep vein thrombosis, cardiac cath, MI, DIC S/S:
hemorrhage, heparin induced thrombocytopenia, toxicity/overdose Common meds- warfarin
(coumadin) {admin once daily, avoid NSAIDs and aspirin}, enoxaparin (lovenox)
251. -ase
Answer: Thrombolytic dissolves clots TX: acute MI, DVT, massive PE, ischemic stroke S/S:
serious bleeding risks from recent wounds, puncture sites, weakened vessels, hypotension NI:
Must take 4-6 hrs of onset Common meds- alteplase (activase, tPA)
252. -asone solone-onide PredCort
Answer: Corticosteroid prevent inflammatory response S/S: Hyperglycemia, peptic ulcer,
fluid retention (increased appetite), withdrawal symptoms, euphoria, insomnia, psychotic
behavior NI: admin w/ meals, DO NOT take with NSAIDS, teach DO NOT stop abruptly
Common meds- prednisone (deltasone), betamethasone (celestone), hydrocortisone sodium
succinate (Solu-cortef), Methylprednisolone sodium succinate (solu-medrol), fluticasone
propionate (advair, flovent) Antiemtic
253. -azine-setron

Answer: reduce N & V S/S: drowsiness, anticholenergic effects, restlessness, tardive
dyskinesia, EPS NI: monitor VS Common meds- promethazine (phenergan),
metaoclopramide (reglan), ondansertron (zofran)
254. (biguanide) Metformin
Answer: Oral hypoglycemic Used in conjunction with diet & exercise; type II NI: teach s/s
of hypoglycemia, HbA1C metformin (glucophage): withhold 48 hrs before/after test w/
contrast
255. catopril, lisinopril, enalapril (vastotec)
Answer: Block the conversion of angiotensin I to angiotensin II TX: HTN, HF, MI, diabetic
nephropathy S/S: Anigoedema, Cough, Electrolyte imbalance (k+) NI: Monitor K+ levels, BP
256. -cillin
Answer: Penicillin TX: pneumonia, upper respiratory infections, septicemia, endocarditis,
rheumatic fever, GYN infectionsNI: hypersensitivity w/ poss. Anaphylaxis
257. -cycline, -floxacin
Answer: Antibiotic
258. Digoxin side effects
Answer: -Fatigue
-Bradycardia
-Anorexia
-Nausea/Vomiting
259. diphenhydramine (benadryl), loratadine (claratin), cetirizine (zyrtec), fexofenadrine
(allegra)
Answer: S/S: anticholenergic effects (cant see, spit, pee, poop), drowsiness NI: use
cautiously pts w/ HTN, PUD, urinary retention, assess hypokalemia, BP, Advise to take @
night
260. -dipine

Answer: Ca+ channel blocker Slows movement of calcium into smooth muscle= arterial
dilation & decreased BP Tx: angina, HTN (verapamil & diltiazem may be used for AFIB, A
flutter, SVT S/S: Constipation, reflex tachycardia, peripheral edema, toxicity Common medsnifedipine (procardia), verapamil, diltiazem
261. Duloxetine (Cymbalta) Fluoxetine (Provac)Escitalopram (Lexapro) Sertraline (Zoloft)
Answer: S/S: weight gain, fatigue, sexual dysfunction, drowsiness NI: avoid alcohol, do not
discontinue abrubptly, monitor for serotonin syndrome! (agitation, confusion, hallucinations)
within first 72 hrs
262. Food to avoid when taking Lithium
Answer: -salty foods
-alcoholic beverages
263. Fosomax
Answer: same as-Alendronate is used for treating osteoporosis in men and postmenopausal
women.
264. Garamycin
Answer: Antibiotic that is toxic to the kidney, injected for radiology studies.
265. gentamicin sulfate
Answer: TX: pneumonia, meningitis, septicemia NI: high risk for ototoxicity, nephrotoxicity,
monitor creatinine & BUN
266. -gliptin -glitazone
Answer: Diabetes Mellitus
267. Haldol-inform if you are taking ____________ medication.
Answer: -benzodiazepine class of anti-anxiety drugs (all ending with "pam") and even,
Xanax.
268. HPV vaccine

Answer: • Human Papilloma Virus (HPV2, HPV4) - -Three doses should be given over a 6
month
• interval for females at 11 to 12 years of age (minimum age is 9 years).
• The second dose should be administered 2 months after the first dose, and the third dose
should be administered 6 months after the first dose.
• HPV4 may be given to males starting at age 9 years of age.
269. ide
Answer: Oral hypoglycemic Used in conjunction with diet & exercise; type II NI: teach s/s
of hypoglycemia, HbA1C metformin (glucophage): withhold 48 hrs before/after test w/
contrast
270. iprazole, apine, idone
Answer: Second Generation Antipsychotic (SGA)
271. Know about Transdermal patch
Answer: • Apply at the same time once each day, preferably in the morning. Keep patch on
for 12 to 14 hr each day.
• Remove the patch at night to reduce the risk of developing tolerance to nitroglycerin. Be
medication-free a minimum of 10 to 12 hr each day (usually at night).
• Do not cut patches to ensure appropriate dosage.
• Place the patch on a hairless area of skin (chest, back, or abdomen) and rotate sites to
prevent skin irritation.
• Wash skin with soap and water and dry thoroughly before applying new patch.
272. Labs for patients taking hydrothiazide
Answer: Periodic determination of serum electrolytes to detect possible electrolyte
imbalance should be done at appropriate intervals.
273. Lipitor
Answer: -lowers cholesterol in blood, "statins". Reduce LDL and total cholesterol. Raise
HDL.
274. Lisinopril therapeutic effect

Answer: blood pressure answer (e.g. 120/80)
275. Lithium report immediately
Answer: slurred speech
276. lorazepam
Answer: TX: Sedative-hypnotics for sleep, Adjuncts to anesthesia to induce relaxation and
amnesia (procedural memory loss), To reduce anxiety (anxiolytic), Panic disorders, To treat
or prevent seizures, For alcohol withdrawal, Muscle relaxant
277. lovastatin (mevacor)
Answer: aid in lowering LDL & increasing HDL S/S: muscle aches, hepatotoxicity,
myopathy, rhabdomyolysis, peripheral neuropathy NI: take in evening, monitor renal and
liver function, low fat/high fiber diet, drug interactions: digoxin, warfarin, NSAIDs, etc.
278. Macrodantin medication
Answer: used to treat or prevent certain urinary tract infections
279. Medication for Schizophrenia
Answer: risperidone, Risperdal
280. metropolol, labetalol, propanolol
Answer: inhibit stimulation of receptor sites= decreased cardiac excitability, CO, myocaridal
O2 demand, lower BP by decreasing release of renin in the kidney TX: HTN, angina,
tachydysryhmias, HF, MI S/S: Bradycardia, Bradypena, Bronchospasms, decreased BP NI:
Monitor DM for hypoglycemia
281. -mycin
Answer: Aminoglycoside (Antimicrobials) TX: pneumonia, meningitis, septicemia NI: high
risk for ototoxicity, nephrotoxicity, monitor creatinine & BUN Common meds- gentamicin
sulfate (garamycin) therapeutic range: 4-12mcg/dL
282. NEUPOGEN (filgrastim)- what is the appropriate route of this med?
Answer: administered by subcutaneous injection or IV infusion

283. nifedipine (procardia), verapamil, diltiazem
Answer: Slows movement of calcium into smooth muscle= arterial dilation & decreased BP
Tx: angina, HTN (verapamil & diltiazem may be used for AFIB, A flutter, SVT S/S:
Constipation, reflex tachycardia, peripheral edema, toxicity
284. -olol
Answer: Beta Blocker inhibit stimulation of receptor sites= decreased cardiac excitability,
CO, myocaridal O2 demand, lower BP by decreasing release of renin in the kidney TX: HTN,
angina, tachydysryhmias, HF, MI S/S: Bradycardia, Bradypena, Bronchospasms, decreased
BP NI: Monitor DM for hypoglycemia Common meds- metropolol, labetalol, propranolol
285. omepazole (prilosec)
Answer: S/S: D,V, N, can increase risk for fractures,, pneumonia, & acid rebound NI: DO
NOT crush, chew, break, notify PROVIDER if GI bleeding!
286. Opioid toxicity-what to check first
Answer: oxygen saturation
287. -pam, -lam
Answer: Benzodiazipines TX: Sedative-hypnotics for sleep, Adjuncts to anesthesia to induce
relaxation and amnesia (procedural memory loss), To reduce anxiety (anxiolytic), Panic
disorders, To treat or prevent seizures, For alcohol withdrawal, Muscle relaxant
288. Patient identifiers
Answer: -Medical record number
-home telephone number
289. Patient reports IV discomfort, what is your first action?
Answer: color and temperature
290. Penicillin
Answer: TX: pneumonia, upper respiratory infections, septicemia, endocarditis, rheumatic
fever, GYN infections NI: hypersensitivity w/ poss. Anaphylaxis

291. -phylline, -terol
Answer: Bronchodilator S/S: tachcardia, palpitations, tremors Common meds- albeuterol
292. -pram, -ine
Answer: SSRIs S/S: weight gain, fatigue, sexual dysfunction, drowsiness NI: avoid alcohol,
do not discontinue abrubptly, monitor for serotonin syndrome! (agitation, confusion,
hallucinations) within first 72 hrs
293. -prazole
Answer: Proton pump inhibitor S/S: D,V, N, can increase risk for fractures,, pneumonia, &
acid rebound NI: DO NOT crush, chew, break, notify PROVIDER if GI bleeding! Common
meds- omepazole (prilosec)
294. Prednisone report
Answer: sore throat
295. pril
Answer: ACE inhibitor Block the conversion of angiotensin I to angiotensin II TX: HTN,
HF, MI, diabetic nephropathy S/S: Anigoedema, Cough, Electrolyte imbalance (k +) NI:
Monitor K+ levels, BP Common med- catopril, lisinopril, enalapril (vastotec)
296. promethazine (phenergan), metaocloprami de (reglan), ondansertron (zofran)
Answer: reduce N & V S/S: drowsiness, anticholenergic effects, restlessness, tardive
dyskinesia, EPS NI: monitor VS
297. RBC Blood transfusion
Answer: Red blood cell (RBC) transfusion is commonly used to increase oxygen-carrying
capacity in patients with anemia or acute blood loss. (prime with normal saline and infuse
with sodium chloride).
298. rednisone (deltasone), betamethasone (celestone), hydrocortisone sodium succinate
(Solu-cortef), Methylprednisol one sodium succinate (solu-medrol), fluticasone propionate
(advair, flovent)

Answer: prevent inflammatory response S/S: Hyperglycemia, peptic ulcer, fluid retention
(increased appetite), withdrawal symptoms, euphoria, insomnia, psychotic behavior NI:
admin w/ meals, DO NOT take with NSAIDS, teach DO NOT stop abruptly
299. sildenafil (viagra)
Answer: s/s: headache, heartburn, diarrhea, flushing, nosebleeds, parathesias, changes in
color vision Contradicted in clients taking nitrates, anticoags, anti HTN
300. Singulair
Answer: used before exercise to prevent breathing problems during exercise
(bronchospasm).
301. -statin
Answer: Antilipidemic aid in lowering LDL & increasing HDL S/S: muscle aches,
hepatotoxicity, myopathy, rhabdomyolysis, peripheral neuropathy NI: take in evening,
monitor renal and liver function, low fat/high fiber diet, drug interactions: digoxin, warfarin,
NSAIDs, etc. Common meds- lovastatin (mevacor)
302. Sumatriptan (treats migraine headaches) adverse effect
Answer: pain, tightness, pressure, or heaviness in the chest, throat, neck, and/or jaw slow or
difficult speech
303. -tidine
Answer: Antiulcer S/S: lethargy, depression, confusion, decreased libido Common medsranitidine hydrochloride (zantac), cimetidine (tagamet), famotidine (pepcid)
304. -tyline
Answer: Tricyclic antidepressant S/S: anticholenergic effects, sedation, toxicity NI: DO NOT
admin with MAOIs, avoid alcohol, contradicted in clients w/ seizures Common medsamitripytyline (elavil)
305. Valporic Acid lab
Answer: liver

306. -vir
Answer: Antiviral
307. warfarin (coumadin) {admin once daily, avoid NSAIDs & aspirin}, enoxaparin
(lovenox)
Answer: inhibit clotting factors (warfarin = factors VII, IX, X) TX: evolving stroke,
pulmonary embolism, massive deep vein thrombosis, cardiac cath, MI, DIC S/S: hemorrhage,
heparin induced thrombocytopenia, toxicity/overdose
308. What food should you increase when taking Lasix?
Answer: -increased amounts of potassium-rich foods (e.g., bananas, prunes, raisins, and
orange juice)
309. What lab values should a nurse monitor for a patient with chronic renal failure?
Answer: • Urinalysis
• Hematuria, proteinuria, and alterations in specific gravity
• Serum creatinine
• Gradual increase of 1 to 2 mg/dL per every 24 to 48 hr for acute renal failure (ARF)
• Gradual increase over months to years for chronic renal failure (CRF) exceeding 4 mg/dL
• Blood urea nitrogen (BUN)
• 80 to 100 mg/dL within 1 week with ARF
• Gradual increase with elevated serum creatinine over months to years for CRF
• 180-200 mg/dL with (CRF)
• Serum electrolytes
• Decreased sodium (dilutional) and calcium, increased potassium, phosphorus, and
magnesium
• Complete blood count (CBC)
• Decreased hemoglobin
310. What medication to administer with Tylenol overdose?
Answer: acetylcysteine (Mucomyst) must be given IV
311. What to understand about Parkinson's Meds?
Answer: they don't cure disease, they slow the process.

312. -zine
Answer: Antihistamine S/S: anticholenergic effects (cant see, spit, pee, poop), drowsiness
NI: use cautiously pts w/ HTN, PUD, urinary retention, assess hypokalemia, BP, Advise to
take @ night Common meds- diphenhydramine (benadryl), loratadine (claratin), cetirizine
(zyrtec), fexofenadrine (allegra)
313. -zosin
Answer: HTN/Prostate

Document Details

Related Documents

person
Mia Robinson View profile
Close

Send listing report

highlight_off

You already reported this listing

The report is private and won't be shared with the owner

rotate_right
Close
rotate_right
Close

Send Message

image
Close

My favorites

image
Close

Application Form

image
Notifications visibility rotate_right Clear all Close close
image
image
arrow_left
arrow_right